INSIGHTSIAS SIMPLYFYING IAS EXAM PREPARATION

INSTA 75 Days REVISION PLAN

UPSC Prelims 2020

INSTA Tests 33 to 36 (GS)

KEY & EXPLANATIONS

www.insightsonindia.com prelims.insightsonindia.com | mains.insightsonindia.com Telegram: insightsIAStips | FB: insightsonindia | TW: vinaygb | YT: INSIGHTS IAS BENGALURU | DELHI | HYDERABAD

Copyright © by Insights IAS All rights are reserved. No part of this document may be reproduced, stored in a retrieval system or transmitted in any form or by any means, electronic, mechanical, photocopying, recording or otherwise, without prior permission of Insights IAS.

Insta 75 Days Revision Plan for UPSC Civil Services Prelims – 2020

This document is the compilation of 100 questions that are part of InsightsIAS famous INSTA REVISION initiative for UPSC civil services Preliminary examination – 2020 (which has become most anticipated annual affair by lakhs of IAS aspirants across the country). These questions are carefully framed so as to give aspirants tough challenge to test their knowledge and at the same time improve skills such as intelligent guessing, elimination, reasoning, deduction etc – which are much needed to sail through tough Civil Services Preliminary Examination conducted by UPSC.

These questions are based on this INSTA Revision Plan which is posted on our website (www.insightsonindia.com). Every year thousands of candidates follow our revision timetable – which is made for SERIOUS aspirants who would like to intensively revise everything that’s important before the exam.

Those who would like to take up more tests for even better preparation, can enroll to Insights IAS Prelims Mock Test Series – 2020 (https://prelims.insightsonindia.com). Every year toppers solve our tests and sail through UPSC civil services exam. Your support through purchase of our tests will help us provide FREE content on our website seamlessly.

Wish you all the best!

Team InsightsIAS

INSTA 75 Days REVISION PLAN for Prelims 2020 - InstaTests

DAY – 33 (InstaTest-33)

1. Consider the following statements regarding National Policy on Biofuels -2018 1. It envisages an indicative target of 10% blending of ethanol in petrol and 5% blending of bio-diesel in diesel. 2. It expands the scope of raw material for ethanol production. 3. To ensure food security, the policy does not allow use of food grains for production of ethanol. Which of the statements given above is/are correct? (a) 1 and 2 only (b) 1 only (c) 2 and 3 only (d) 2 only

Solution: D

The National Policy on Biofuels-2018 approved by the Government envisages an indicative target of 20% blending of ethanol in petrol and 5% blending of bio-diesel in diesel by 2030. National Policy on biofuels- salient features: 1. Categorization: The Policy categorises biofuels as “Basic Biofuels” viz. First Generation (1G) bioethanol & biodiesel and “Advanced Biofuels” – Second Generation (2G) ethanol, Municipal Solid Waste (MSW) to dropin fuels, Third Generation (3G) biofuels, bio-CNG etc. to enable extension of appropriate financial and fiscal incentives under each category. 2. Scope of raw materials: The Policy expands the scope of raw material for ethanol production by allowing use of Sugarcane Juice, Sugar containing materials like Sugar Beet, Sweet Sorghum, Starch containing materials like Corn, Cassava, Damaged food grains like wheat, broken rice, Rotten Potatoes, unfit for human consumption for ethanol production. 3. Protection to farmers: Farmers are at a risk of not getting appropriate price for their produce during the surplus production phase. Taking this into account, the Policy allows use of surplus food grains for production of ethanol for blending with petrol with the approval of National Biofuel Coordination Committee 4. Viability gap funding: With a thrust on Advanced Biofuels, the Policy indicates a viability gap funding scheme for 2G ethanol Bio refineries of Rs.5000 crore in 6 years in addition to additional tax incentives, higher purchase price as compared to 1G biofuels.

www.insightsonindia.com 1 Insights IAS INSTA 75 Days REVISION PLAN for Prelims 2020 - InstaTests

5. Boost to biodiesel production: The Policy encourages setting up of supply chain mechanisms for biodiesel production from non-edible oilseeds, Used Cooking Oil, short gestation crops.

2. Consider the following statements regarding Lala Lajpat Rai 1. He founded the Indian Home Rule League of America in New York City in 1917. 2. He died at Lahore in 1928 after he was attacked by police during a protest rally against the Simon Commission. 3. Kesari and New were the important works of him. Which of the statements given above is/are correct? (a) 1 and 2 only (b) 2 and 3 only (c) 1 and 3 only (d) 1, 2 and 3

Solution: A

Lala Lajpat Rai

• Lala Lajpat Rai is remembered for his role during the Swadeshi movement and for his advocacy of education. He became a follower of Dayanand Saraswati, the founder of the Arya Samaj, and went on to become one of the society’s leaders. He also helped found the Punjab National Bank. • In 1885, Rai established the Dayanand Anglo-Vedic School in Lahore and remained a committed educationist throughout his life. • Rai, Tilak, and Bipin Chandra Pal (called Lal-Bal-Pal) fervently advocated the use of Swadeshi goods and mass agitation in the aftermath of the controversial Partition of in 1905 by Lord Curzon. • He founded the Indian Home Rule League of America in New York City in 1917. • He was elected President of the Indian National Congress during its Special Session in Kolkata in 1920, which saw the launch of Mahatma Gandhi’s Non-cooperation Movement. • The patriot died at Lahore in 1928 after he was attacked by police during a protest rally against the Simon Commission. • His important works include: ‘The Arya Samaj’, ‘Young India’, ‘England’s Debt to India’, ‘Evolution of Japan’, ‘India’s Will to Freedom’, ‘Message of the Bhagwad Gita’, ‘Political Future of India’, ‘Problem of National Education in India’, ‘The Depressed Glasses’, and the travelogue ‘United States of America’.

www.insightsonindia.com 2 Insights IAS INSTA 75 Days REVISION PLAN for Prelims 2020 - InstaTests

3. Consider the following statements regarding Anti-Satellite Missile in India 1. It’s essentially a missile that can destroy or jam an enemy country’s satellite in space. 2. It is a ISRO-developed Ballistic Missile Defence (BMD) Interceptor Missile which successfully engaged an Indian orbiting target satellite in Low Earth Orbit (LEO) Which of the statements given above is/are correct? (a) 1 only (b) 2 only (c) Both 1 and 2 (d) Neither 1 nor 2

Solution: A

Anti-Satellite (ASAT) Missile

• It’s essentially a missile that can destroy or jam an enemy country’s satellite in space. • It is a DRDO-developed Ballistic Missile Defence (BMD) Interceptor Missile which successfully engaged an Indian orbiting target satellite in Low Earth Orbit (LEO) in a ‘Hit to Kill’ mode. https://www.thehindu.com/news/national/anti-satellite-missile-capability-showcased-in-r- day-parade/article30657451.ece

4. Consider the following statements regarding Pt. Madan Mohan Malaviya 1. He had not served as the President of the Indian National Congress. 2. He was the editor of a Hindi magazine, ‘Hindosthan’. 3. He was a social reformer who opposed untouchability and worked for the temple entry of Dalits. Which of the statements given above is/are correct? (a) 1 and 2 only (b) 2 and 3 only (c) 1 and 3 only (d) 1, 2 and 3

Solution: B

www.insightsonindia.com 3 Insights IAS INSTA 75 Days REVISION PLAN for Prelims 2020 - InstaTests

Pt. Madan Mohan Malaviya

• Pt. Madan Mohan Malaviya was a freedom fighter and social reformer. He had served as the President of the INC on four occasions. He was awarded the Bharat Ratna posthumously in 2014. • He was the editor of a Hindi magazine, ‘Hindosthan’. He became the editor of the ‘Indian Opinion’ in 1889. He also started a Hindi weekly ‘Abhyudaya’, an English daily ‘Leader’, a Hindi newspaper ‘Maryada’. • Pandit Malaviya was instrumental in founding the Benares Hindu University in 1916. He also became its Vice-Chancellor till 1939. • He was opposed to separate electorates to and the Lucknow Pact. • He was also against the INC’s participation in the Khilafat Movement. • He was a participant in the Second Round Table Conference in 1931. • He started the Ganga Mahasabha to oppose construction of dams in the Ganga. • He was also a social reformer who opposed untouchability. He worked for the temple entry of Dalits at the Kalaram Temple at Nashik, Maharashtra. • He also founded the organisation Shri Mathura Vrindavan Hasanand Gochar Bhoomi in Vrindavan.

5. Periyar E. V. Ramasamy was associated with which of following movements/Parties? 1. Vaikom Satyagraha 2. Justice Party 3. Self-Respect Movement 4. Dravidar Kazhagam Select the correct answer using the code given below (a) 1, 2 and 3 only (b) 2, 3 and 4 only (c) 1, 3 and 4 only (d) 1, 2, 3 and 4

Solution: D

Periyar E. V. Ramasamy

• Born in 1879, Periyar is remembered for the Self Respect Movement to redeem the identity and self-respect of Tamils. He envisaged a Dravida homeland of Dravida Nadu, and launched a political party, Dravidar Kazhagam (DK). • He associated himself with the Justice Party and the Self Respect Movement.

www.insightsonindia.com 4 Insights IAS INSTA 75 Days REVISION PLAN for Prelims 2020 - InstaTests

• Periyar’s fame spread beyond the Tamil region during the Vaikom Satyagraha of 1924, a mass movement to demand that lower caste persons be given the right to use a public path in front of the famous Vaikom temple. • In the 1940s, Periyar launched Dravidar Kazhagam, which espoused an independent Dravida Nadu comprising Tamil, Malayalam, Telugu, and Kannada speakers. • As a social reformer, he focused on social, cultural and gender inequalities, and his reform agenda questioned matters of faith, gender and tradition. He asked people to be rational in their life choices. He argued that women needed to be independent, not mere child-bearers, and insisted that they be allowed an equal share in employment.

6. Operation Vanilla, sometime seen in the news, is (a) To assist the population of Australia affected by Bush fires (b) To assist the stranded Indians in the city of Wuhan (c) To assist the population of Madagascar affected by Cyclone Diane (d) None of the statements (a), (b) and (c) are correct.

Solution: C

• Operation Vanilla is launched by Indian Navy in the Southern Indian Ocean. The Operation has been launched to assist the population of Madagascar that were affected by Cyclone Diane. https://www.thehindu.com/news/national/indian-navy-launches-operation-vanilla-to-help- flood-hit-madagascar/article30677388.ece

7. Consider the following statements regarding Indian National Congress 1. The first session of the Indian National Congress was presided over by Womesh Chandra Bonnerjee. 2. The first session of the Indian National Congress was held in Calcutta in December 1885. 3. Kadambini Ganguly was the first woman president to address the Congress session. Which of the statements given above is/are correct? (a) 1 only (b) 2 and 3 only (c) 1 and 3 only www.insightsonindia.com 5 Insights IAS INSTA 75 Days REVISION PLAN for Prelims 2020 - InstaTests

(d) 1, 2 and 3

Solution: A

• The final shape to the of all-India organisation idea was given by a retired English civil servant, A.O. Hume, who mobilised leading intellectuals of the time and, with their cooperation, organised the first session of the Indian National Congress at Gokuldas Tejpal Sanskrit College in Bombay in December 1885. • As a prelude to this, two sessions of the Indian National Conference had been held in 1883 and 1885, which had representatives drawn from all major towns of India. Surendranath Banerjea and Ananda Mohan Bose were the main architects of the Indian National Conference. • The first session of the Indian National Congress was attended by 72 delegates and presided over by Womesh Chandra Bonnerjee. • In 1890, Kadambini Ganguly, the first woman graduate of Calcutta University, addressed the Congress session, which symbolised the commitment of the freedom struggle to give the women of India their due status in national life.

8. Consider the following statements regarding Chabahar port 1. The port provides access to the resources and markets of landlocked Afghanistan and Central Asia bypassing Pakistan. 2. Chabahar port will be linked to International North-South Transport Corridor (INSTC) Which of the statements given above is/are correct? (a) 1 only (b) 2 only (c) Both 1 and 2 (d) Neither 1 nor 2

Solution: C

• The Chabahar port project will be the first overseas venture for Indian state-owned ports such as JNPT and the Deendayal Port Trust. It is located in the Sistan- Baluchistan Province on Iran’s South-eastern coast (outside the Persian Gulf). • The port provides access to the resources and markets of landlocked Afghanistan and Central Asia bypassing Pakistan. www.insightsonindia.com 6 Insights IAS INSTA 75 Days REVISION PLAN for Prelims 2020 - InstaTests

• Chabahar port will be linked to International North-South Transport Corridor (INSTC), thus serve as India’s outreach in region to Afghanistan and beyond to Russia, Eastern Europe.

9. Which of the following resolutions was/were discussed in Surat session, which eventually led to the split of Congress? 1. Swadeshi 2. Boycott 3. National Education Select the correct answer using the code given below (a) 1 and 2 only (b) 2 and 3 only (c) 1 and 3 only (d) 1, 2 and 3

Solution: D

• The Congress split at Surat came in December 1907, around the time when revolutionary activity had gained momentum. • The two events were not unconnected. In December 1905, at the Benaras session of the Indian National Congress presided over by Gokhale, the Moderate-Extremist differences came to the fore. www.insightsonindia.com 7 Insights IAS INSTA 75 Days REVISION PLAN for Prelims 2020 - InstaTests

• The Extremists wanted a strong resolution supporting their programme at the Benaras session. The Moderates, on the other hand, were not in favour of extending the movement beyond Bengal and were totally opposed to boycott of councils and similar associations. • At the Calcutta session of the Congress in December 1906, the Moderate enthusiasm had cooled a bit because of the popularity of the Extremists and the revolutionaries and because of communal riots. Here, the Extremists wanted either Tilak or Lajpat Rai as the president, while the Moderates proposed the name of Dadabhai Naoroji, who was widely respected by all the nationalists. • The Extremists wanted the 1907 session to be held in Nagpur (Central Provinces) with Tilak or Lajpat Rai as the president along with a reiteration of the swadeshi, boycott and national education resolutions. The Moderates wanted the session at Surat in order to exclude Tilak from the presidency, since a leader from the host province could not be session president (Surat being in Tilak’s home province of Bombay). • Instead, they wanted Rashbehari Ghosh as the president and sought to drop the resolutions on swadeshi, boycott and national education. Both sides adopted rigid positions, leaving no room for compromise.

10. Consider the following statements regarding Anushilan Samiti 1. It was founded by Promotha Mitter. 2. Yugantar was the weekly publication of Anushilan Samiti Which of the statements given above is/are correct? (a) 1 only (b) 2 only (c) Both 1 and 2 (d) Neither 1 nor 2

Solution: C

• By the 1870s, Calcutta’s student community was honeycombed with secret societies, but these were not very active. The first revolutionary groups were organised in 1902 in Midnapore (under Jnanendranath Basu) and in Calcutta (the Anushilan Samiti founded by Promotha Mitter, and including Jatindranath Banerjee, Barindra Kumar Ghosh and others.) • But their activities were limited to giving physical and moral training to the members and remained insignificant till 1907-08.

www.insightsonindia.com 8 Insights IAS INSTA 75 Days REVISION PLAN for Prelims 2020 - InstaTests

• In April 1906, an inner circle within Anushilan (Barindra Kumar Ghosh, Bhupendranath Dutta) started the weekly Yugantar and conducted a few abortive ‘actions’.

11. Consider the following statements regarding Komagata Maru Incident 1. Komagata Maru was the name of the Japanese steamship which was carrying the passengers. 2. It was carrying passengers from India to Vancouver and were turned back by Canadian authorities. Which of the statements given above is/are correct? (a) 1 only (b) 2 only (c) Both 1 and 2 (d) Neither 1 nor 2

Solution: A

Komagata Maru Incident

• The importance of this event lies in the fact that it created an explosive situation in the Punjab. Komagata Maru was the name of a ship which was carrying 370 passengers, mainly Sikh and Punjabi Muslim would-be immigrants, from Singapore to Vancouver. • They were turned back by Canadian authorities after two months of privation and uncertainty. It was generally believed that the Canadian authorities were influenced by the British government. The ship finally anchored at Calcutta in September 1914. The inmates refused to board the Punjab bound train. In the ensuing conflict with the police at Budge Budge near Calcutta, 22 persons died. • The Komagata Maru incident involved the Japanese steamship Komagata Maru, on which a group of people from British India attempted to emigrate to Canada in 1914, but most were denied entry and forced to return to Budge Budge Calcutta (present- day Kolkata), India.

12. Consider the following statements regarding Home Rule Movement 1. It was the Indian response to the First World War. 2. Indian Home Rule Leagues were organised on the lines of the Russian Home Rule Leagues.

www.insightsonindia.com 9 Insights IAS INSTA 75 Days REVISION PLAN for Prelims 2020 - InstaTests

3. Besant got the approval from Congress to setup Home Rule Leagues and many congress leaders joined the movement. Which of the statements given above is/are correct? (a) 1 only (b) 2 and 3 only (c) 1 and 3 only (d) 1, 2 and 3

Solution: A

The Home Rule Movement was the Indian response to the First World War in a less charged but a more effective way than the response of Indians living abroad which took the form of the romantic Ghadr adventure.

• Two Indian Home Rule Leagues were organised on the lines of the Irish Home Rule Leagues and they represented the emergence of a new trend of aggressive politics. • Annie Besant and Tilak were the pioneers of this new trend. Although Besant failed to get the Congress to approve her scheme of Home Rule Leagues, the Congress did commit itself to a programme of educative propaganda and to a revival of local-level Congress committees. • Not willing to wait for too long, Besant laid the condition that if the Congress did not implement its commitments, she would be free to set up her own league—which she finally had to, as there was no response from the Congress. • Tilak and Besant set up their separate leagues to avoid any friction. • Tilak set up his Home Rule League in April 1916 and it was restricted to Maharashtra (excluding Bombay city), Karnataka, Central Provinces and Berar. It had six branches and the demands included swarajya, formation of linguistic states and education in the vernacular.

13. Which of the following was/were the reforms under Government of India Act, 1919 1. Women were given the right to vote. 2. A bicameral arrangement was introduced. 3. The legislative councils could initiate legislation Select the correct answer using the code given below (a) 1 only (b) 2 and 3 only

www.insightsonindia.com 10 Insights IAS INSTA 75 Days REVISION PLAN for Prelims 2020 - InstaTests

(c) 1 and 2 only (d) 1, 2 and 3

Solution: D

In line with the government policy contained in Montagu’s statement of August 1917, the government announced further constitutional reforms in July 1918, known as Montagu- Chelmsford or Montford Reforms. Based on these, the Government of India Act, 1919 was enacted. Legislature 1) Provincial legislative councils were further expanded and 70 per cent of the members were to be elected. 2) The system of communal and class electorates was further consolidated. 3) Women were also given the right to vote. 4) The legislative councils could initiate legislation but the governor’s assent was required. The governor could veto bills and issue ordinances. 5) The legislative councils could reject the budget but the governor could restore it, if necessary. 6) The legislators enjoyed freedom of speech. Legislature 1) A bicameral arrangement was introduced. 2) The Council of State had a tenure of 5 years and had only male members, while the Central Legislative Assembly had a tenure of 3 years. 3) The legislators could ask questions and supplementaries, pass adjournment motions and vote a part of the budget, but 75 per cent of the budget was still not votable.

14. Which of the following is/are considered as Contempt of Court? 1. Disobeying court orders, 2. Interfering with judicial proceedings, 3. Obstructing the administration of justice 4. Scandalizing or lowering the authority of the court Select the correct answer using the code given below (a) 1, 2 and 3 only (b) 2, 3 and 4 only (c) 1, 2 and 4 only (d) 1, 2, 3 and 4 www.insightsonindia.com 11 Insights IAS INSTA 75 Days REVISION PLAN for Prelims 2020 - InstaTests

Solution: D

• The Supreme Court has held that courts are empowered to order parties in a contempt case to surrender their passport in order to ensure their presence in the proceedings. Background:

• The court was hearing a contempt case in connection with a civil suit on partition of a disputed property in Delhi. What is contempt under the Indian law? In India, the Contempt of Courts Act, 1971, divides contempt into civil contempt and criminal contempt. 1. ‘Civil contempt’ is a ‘wilful disobedience to any judgment, decree, direction, order, writ or other processes of a Court or wilful breach of an undertaking given to the court’. 2. ‘Criminal contempt’ is ‘the publication (whether by words, spoken or written, or by signs, or by visible representation, or otherwise) of any matter or the doing of any other act whatsoever which: 1) Scandalises or tends to scandalise, or lowers or tends to lower the authority of, any court. 2) Prejudices, or interferes or tends to interfere with the due course of any judicial proceeding. 3) Interferes or tends to interfere with, or obstructs or tends to obstruct, the administration of justice in any other manner.’ Contempt of Courts (Amendment) Act, 2006:

• The statute of 1971 has been amended by the Contempt of Courts (Amendment) Act, 2006 to include the defence of truth under Section 13 of the original legislation. • Section 13 that already served to restrict the powers of the court in that they were not to hold anyone in contempt unless it would substantially interfere with the due process of justice, the amendment further states that the court must permit ‘justification by truth as a valid defence if it is satisfied that it is in public interest and the request for invoking the said defence is bona fide.’ Constitutional Background:

• Article 129: Grants Supreme Court the power to punish for contempt of itself. • Article 142(2): Enables the Supreme Court to investigate and punish any person for its contempt. • Article 215: Grants every High Court the power to punish for contempt of itself.

www.insightsonindia.com 12 Insights IAS INSTA 75 Days REVISION PLAN for Prelims 2020 - InstaTests

15. Consider the following statements regarding Convalescent Plasma Therapy (CPT) 1. It seeks to make use of the antibodies developed from genetic engineering technique. 2. Therapy is useful in treating the patients infected with corona and other type of viruses. Which of the statements given above is/are correct? (a) 1 only (b) 2 only (c) Both 1 and 2 (d) Neither 1 nor 2

Solution: B

• It seeks to make use of the antibodies developed in the recovered patient against the coronavirus. • The whole blood or plasma from such people is taken, and the plasma is then injected in critically ill patients so that the antibodies are transferred and boost their fight against the virus. • A study in The Lancet Infectious Diseases said a COVID-19 patient usually develops primary immunity against the virus in 10-14 days. • Therefore, if the plasma is injected at an early stage, it can possibly help fight the virus and prevent severe illness. https://indianexpress.com/article/explained/recovered-covid-19-patients-plasma-what-is- this-experimental-therapy-6337764/

16. Consider the following statements regarding 1. Gandhi was requested by Rajkumar Shukla to look into the problems of the farmers in Champaran. 2. It was a no-tax peasant struggle and demand for plague bonus. 3. The government appointed a committee to go into the matter and nominated Gandhi as a member. Which of the statements given above is/are correct? (a) 1 only (b) 2 and 3 only (c) 1 and 3 only

www.insightsonindia.com 13 Insights IAS INSTA 75 Days REVISION PLAN for Prelims 2020 - InstaTests

(d) 1, 2 and 3

Solution: C

Champaran Satyagraha (1917)—First Civil Disobedience

• Gandhi was requested by Rajkumar Shukla, a local man, to look into the problems of the farmers in context of indigo planters of Champaran in Bihar. The European planters had been forcing the peasants to grow indigo on 3/20 part of the total land (called tinkathia system). • When towards the end of the nineteenth century German synthetic dyes replaced indigo, the European planters demanded high rents and illegal dues from the peasants in order to maximize their profits before the peasants could shift to other crops. Besides, the peasants were forced to sell the produce at prices fixed by the Europeans. • When Gandhi, joined now by Rajendra Prasad, Mazharul- Haq, Mahadeo Desai, Narhari Parekh, and J.B. Kripalani, reached Champaran to probe into the matter, the authorities ordered him to leave the area at once. • Gandhi defied the order and preferred to face the punishment. This passive resistance or civil disobedience of an unjust order was a novel method at that time. Finally, the authorities retreated and permitted Gandhi to make an enquiry. • Now, the government appointed a committee to go into the matter and nominated Gandhi as a member. Gandhi was able to convince the authorities that the tinkathia system should be abolished and that the peasants should be compensated for the illegal dues extracted from them. As a compromise with the planters, he agreed that only 25 per cent of the money taken should be compensated.

17. Gender Social Norms Index has been released by (a) World Economic Forum (WEF) (b) Organisation of Economic Cooperation and Development (OECD) (c) UN Development Programme (UNDP) (d) United Nations Educational, Scientific and Cultural Organization (UNESCO)

Solution: C

• The first Gender Social Norms Index was recently released by the UN Development Programme (UNDP). www.insightsonindia.com 14 Insights IAS INSTA 75 Days REVISION PLAN for Prelims 2020 - InstaTests

About the index:

• This index measures how social beliefs obstruct gender equality in areas like politics, work, and education, and contains data from 75 countries, covering over 80 percent of the world’s population. • The index found new clues to the invisible barriers women face in achieving equality – potentially forging a path forward to breaking through the so-called “glass ceiling”. Key findings:

• Despite decades of progress closing the equality gap between men and women, close to 90 percent of men and women hold some sort of bias against women. • Almost half of those polled feel that men are superior political leaders. • More than 40 per cent believe they make better business executives and are more entitled to jobs when the economy is lagging. • Moreover, 28 per cent think it is justified for a man to beat his wife. • The analysis also highlighted a bias shift in some 30 countries, revealing that while some show improvements, attitudes in others appear to have worsened in recent years – signaling that progress cannot be taken for granted.

www.insightsonindia.com 15 Insights IAS INSTA 75 Days REVISION PLAN for Prelims 2020 - InstaTests

18. Consider the following statements regarding The Anarchical and Revolutionary Crimes Act 1. It was based on the recommendations made by the Rowlatt Commission. 2. The act allowed political activists to be tried without juries or even imprisoned without trial. 3. The law of habeas corpus was sought to be suspended. Which of the statements given above is/are correct? (a) 1 and 2 only (b) 2 and 3 only (c) 1 and 3 only (d) 1, 2 and 3

Solution: D

• The Anarchical and Revolutionary Crimes Act, popularly known as the Rowlatt Act, was based on the recommendations made in the previous year to the Imperial Legislative Council by the Rowlatt Commission, headed by the British judge, Sir Sidney Rowlatt, to investigate the ‘seditious conspiracy’ of the Indian people. (The committee had recommended that activists should be deported or imprisoned without trial for two years, and that even possession of seditious newspapers would be adequate evidence of guilt.) • All the elected Indian members of the Imperial Legislative Council voted against the bill but they were in a minority and easily overruled by the official nominees. All the elected Indian members—who included Mohammed Ali Jinnah, Madan Mohan Malaviya and Mazhar Ul Haq – resigned in protest. • The act allowed political activists to be tried without juries or even imprisoned without trial. It allowed arrest of Indians without warrant on the mere suspicion of ‘treason’. Such suspects could be tried in secrecy without recourse to legal help. A special cell consisting of three high court judges was to try such suspects and there was no court of appeal above that panel. This panel could even accept evidence not acceptable under the Indian Evidences Act. • The law of habeas corpus, the basis of civil liberty, was sought to be suspended.

19. Which of the following personalities was/ were Swarajists during India’s freedom struggle? 1. Rajendra Prasad 2. C.R. Das 3. Motilal Nehru

www.insightsonindia.com 16 Insights IAS INSTA 75 Days REVISION PLAN for Prelims 2020 - InstaTests

4. Ajmal Khan Select the correct answer using the code given below (a) 1, 2 and 3 only (b) 2, 3 and 4 only (c) 1, 3 and 4 only (d) 1, 2, 3 and 4

Solution: B

• One section led by C.R. Das, Motilal Nehru and Ajmal Khan wanted an end to the boycott of legislative councils so that the nationalists could enter them to expose the basic weaknesses of these assemblies and use these councils as an arena of political struggle to arouse popular enthusiasm. • They wanted, in other words, to ‘end or mend’ these councils, i.e., if the government did not respond to the nationalists’ demands, then they would obstruct the working of these councils. • Those advocating entry into legislative councils came to be known as the ‘Swarajists’, while the other school of thought led by C. Rajagopalachari, Vallabhbhai Patel, Rajendra Prasad and M.A. Ansari came to be known as the ‘Nochangers’. • The ‘No-changers’ opposed council entry, advocated concentration on constructive work, and continuation of boycott and non-cooperation, and quiet preparation for resumption of the suspended civil disobedience programme.

20. Consider the following statements regarding North East Venture Fund (NEVF) 1. It is set up by North Eastern Council in association with Ministry of Development of North Eastern Region (M-DoNER). 2. It is the first dedicated venture capital fund for the North Eastern Region. 3. It aims to provide funding for the tribal women entrepreneurs of North Eastern region. Which of the statements given above is/are correct? (a) 1 and 2 only (b) 1 and 3 only (c) 2 only (d) 1, 2 and 3

www.insightsonindia.com 17 Insights IAS INSTA 75 Days REVISION PLAN for Prelims 2020 - InstaTests

Solution: C

About North East Venture Fund (NEVF):

• Launched in September 2017. • Set up by North Eastern Development Finance Corporation Limited (NEDFi) in association with Ministry of Development of North Eastern Region (M-DoNER). • It is a close ended fund with capital commitment of Rs 100 crore. • It is the first dedicated venture capital fund for the North Eastern Region. • Objective: to contribute to the entrepreneurship development of the NER and achieve attractive risk-adjusted returns through long term capital appreciation by way of investments in privately negotiated equity/ equity related investments. • The investment under this schemer ranges from Rs. 25 lakh to Rs.10 crore per venture, which is long term in nature with investment horizon of 4-5 years.

21. Consider the following statements regarding Mudumalai Tiger Reserve 1. It shares its boundaries with the states of Karnataka and Kerala. 2. It is a part of Nilgiri Biosphere Reserve Which of the statements given above is/are correct? (a) 1 only (b) 2 only (c) Both 1 and 2 (d) Neither 1 nor 2

Solution: C

• The Mudumalai National Park and Wildlife Sanctuary also a declared tiger reserve, lies on the northwestern side of the Nilgiri Hills in Nilgiri District, in Tamil Nadu, India. It shares its boundaries with the states of Karnataka and Kerala. • It is a part of Nilgiri Biosphere Reserve along with Wayanad Wildlife Sanctuary (Kerala) in the West, Bandipur National Park (Karnataka) in the North, Mukurthi National Park and Silent Valley in the South. The sanctuary is divided into five ranges – Masinagudi, Thepakadu, Mudumalai, Kargudi and Nellakota.

www.insightsonindia.com 18 Insights IAS INSTA 75 Days REVISION PLAN for Prelims 2020 - InstaTests

22. Which of the following is/are the recommendations of Nehru Report? 1. Dominion status on lines of self-governing dominions desired by Indians 2. Rejection of separate electorates 3. Linguistic provinces 4. Complete dissociation of State from religion Select the correct answer using the code given below (a) 1, 2 and 3 only (b) 2, 3 and 4 only (c) 1, 3 and 4 only (d) 1, 2, 3 and 4

Solution: D

The Nehru Report confined itself to British India, as it envisaged the future link-up of British India with the princely states on a federal basis. For the dominion it recommended:

www.insightsonindia.com 19 Insights IAS INSTA 75 Days REVISION PLAN for Prelims 2020 - InstaTests

• Dominion status on lines of self-governing dominions as the form of government desired by Indians (much to the chagrin of younger, militant section—Nehru being prominent among them). • Rejection of separate electorates which had been the basis of constitutional reforms so far; instead, a demand for joint electorates with reservation of seats for Muslims. • Linguistic provinces. • Nineteen fundamental rights including equal rights for women, right to form unions, and universal adult suffrage. • Responsible government at the Centre and in provinces • Full protection to cultural and religious interests of Muslims. • Complete dissociation of State from religion.

23. Consider the following statements regarding individual satyagraha 1. The demand of the satyagraha would be the freedom of speech against the war through an anti-war declaration. 2. It was associated with Delhi Chalo Movement. Which of the statements given above is/are correct? (a) 1 only (b) 2 only (c) Both 1 and 2 (d) Neither 1 nor 2

Solution: C

• The government had taken the adamant position that no constitutional advance could be made till the Congress came to an agreement with the Muslim leaders. It issued ordinance after ordinance taking away the freedom of speech and that of the press and the right to organise associations. • Towards the end of 1940, the Congress once again asked Gandhi to take command. Gandhi now began taking steps which would lead to a mass struggle within his broad strategic perspective. He decided to initiate a limited satyagraha on an individual basis by a few selected individuals in every locality. The aims of launching individual satyagraha were— 1. to show that nationalist patience was not due to weakness; 2. to express people’s feeling that they were not interested in the war and that they made no distinction between Nazism and the double autocracy that ruled India; and

www.insightsonindia.com 20 Insights IAS INSTA 75 Days REVISION PLAN for Prelims 2020 - InstaTests

3. to give another opportunity to the government to accept Congress’ demands peacefully. The demand of the satyagraha would be the freedom of speech against the war through an anti-war declaration. If the government did not arrest the satyagrahi, he or she would not only repeat it but move into villages and start a march towards Delhi, thus precipitating a movement which came to be known as the ‘Delhi Chalo Movement’. Vinoba Bhave was the first to offer the satyagraha and Nehru, the second. By May 1941, 25,000 people had been convicted for individual civil disobedience.

24. Consider the following statements regarding Central Pollution Control Board 1. It was established under Air (Prevention and Control of Pollution) Act, 1981. 2. It serves as a field formation and also provides technical services to the Ministry of Environment and Forests under the provisions of the Environment (Protection) Act, 1986. Which of the statements given above is/are correct? (a) 1 only (b) 2 only (c) Both 1 and 2 (d) Neither 1 nor 2

Solution: B

The Central Pollution Control Board (CPCB) of India is a statutory organisation under the Ministry of Environment, Forest and Climate Change (MoEF&CC). It was established in 1974 under the Water (Prevention and Control of pollution) Act, 1974.

• CPCB is entrusted with the powers and functions under the Air (Prevention and Control of Pollution) Act, 1981. It serves as a field formation and also provides technical services to the Ministry of Environment and Forests under the provisions of the Environment (Protection) Act, 1986. It Co-ordinates the activities of the State Pollution Control Boards by providing technical assistance and guidance and also resolves disputes among them.

25. Consider the following statements regarding Desai-Liaqat Pact 1. An equal number of persons nominated by the Congress and the League in the central legislature for the formation of an interim government 2. Nearly 20% of the seats were reserved for minorities. www.insightsonindia.com 21 Insights IAS INSTA 75 Days REVISION PLAN for Prelims 2020 - InstaTests

Which of the statements given above is/are correct? (a) 1 only (b) 2 only (c) Both 1 and 2 (d) Neither 1 nor 2

Solution: C

Desai-Liaqat Pact Desai-Liaqat Pact was concluded between Bhulabhai Desai of the Congress and Liaqat Ali Khan of the Muslim League. It was to find out the way out of the 1942-45 political impasses. Efforts continued to end the deadlock. Bhulabhai Desai, leader of the Congress Party in the Central Legislative Assembly, met Liaqat Ali Khan, deputy leader of the Muslim League in that Assembly, and both of them came up with the draft proposal for the formation of an interim government at the centre, consisting of—

• an equal number of persons nominated by the Congress and the League in the central legislature. • 20% reserved seats for minorities. No settlement could be reached between the Congress and the League on these lines, but the fact that a sort of parity between the Congress and the League was decided upon had far- reaching consequences.

DAY – 34 (InstaTest-34)

26. Consider the following statements regarding Sanyasi Revolt 1. The famine of 1770 and the harsh economic order of the British compelled a group of sanyasis to fight the British. 2. The Muslims didn’t participate in this revolt because of its religious character. 3. Anandamath by Bankim Chandra Chattopadhyay, is based on the Sanyasi Revolt Which of the statements given above is/are correct? (a) 1 and 2 only (b) 2 and 3 only (c) 1 and 3 only (d) 1, 2 and 3 www.insightsonindia.com 22 Insights IAS INSTA 75 Days REVISION PLAN for Prelims 2020 - InstaTests

Solution: C

Sanyasi Revolt (1763-1800)

• The disastrous famine of 1770 and the harsh economic order of the British compelled a group of sanyasis in Eastern India to fight the British yoke. • Originally peasants, even some evicted from land, these sanyasis were joined by a large number of dispossessed small zamindars, disbanded soldiers and rural poor. They raided Company factories and the treasuries, and fought the Company’s forces. It was only after a prolonged action that Warren Hastings could subdue the sanyasis. • Equal participation of Hindus and Muslims characterised the uprisings, sometimes referred to as the Rebellion. Majnum Shah (or ), Chirag Ali, Musa Shah, Bhawani Pathak and Debi Chaudhurani were important leaders. Debi Chaudhurani’s participation recognizes the women’s role in early resistances against the British. • Anandamath, a semi-historical novel by Bankim Chandra Chattopadhyay, is based on the Sanyasi Revolt. Bankim Chandra also wrote a novel, Devi Chaudhurani, as he saw the importance of women too taking up the struggle against an alien rule that posed a threat to traditional Indian values.

27. Consider the following statements regarding Ahom Revolt 1. It was a revolt against the British who attempted to incorporate the Ahom’s’ territories in the Company’s dominion 2. The rebellion was organized under the leadership of Gomdhar Konwar, an Ahom prince. Which of the statements given above is/are correct? (a) 1 only (b) 2 only (c) Both 1 and 2 (d) Neither 1 nor 2

Solution: C

Ahom Revolt (1828)

• The British had pledged to withdraw from Assam after the First Burma War (1824-26). But, after the war, instead of withdrawing, the British attempted to incorporate the Ahoms’ territories in the Company’s dominion. www.insightsonindia.com 23 Insights IAS INSTA 75 Days REVISION PLAN for Prelims 2020 - InstaTests

• This sparked off a rebellion in 1828 under the leadership of Gomdhar Konwar, an Ahom prince, along with compatriots, such as Dhanjoy Bongohain, and Jairam Khargharia Phukan. Assembling near Jorhat, the rebels formally made Gomdhar Konwar the king. • Finally, the Company decided to follow a conciliatory policy and handed over Upper Assam to Maharaja Purandar Singh Narendra and part of the kingdom was restored to the Assamese king.

28. Consider the following statements regarding Kuka Movement 1. The Kuka Movement was founded by Baba Ram Singh. 2. The Kukas wanted to remove the British and restore Sikh rule over Punjab. 3. The concepts of Swadeshi and non-cooperation were propagated by the Kukas. Which of the statements given above is/are correct? (a) 1 and 2 only (b) 2 and 3 only (c) 1 and 3 only (d) 1, 2 and 3

Solution: B

Kuka Movement

• The Kuka Movement was founded in 1840 by Bhagat Jawahar Mal (also called Sian Saheb) in western Punjab. A major leader of the movement after him was Baba Ram Singh. • (He founded the Namdhari Sikh sect.) After the British took Punjab, the movement got transformed from a religious purification campaign to a political campaign. Its basic tenets were abolition of caste and similar discriminations among Sikhs, discouraging the consumption of meat and alcohol and drugs, permission for intermarriages, widow remarriage, and encouraging women to step out of seclusion. • On the political side, the Kukas wanted to remove the British and restore Sikh rule over Punjab; they advocated wearing hand-woven clothes and boycott of English laws and education and products. So, the concepts of Swadeshi and non-cooperation were propagated by the Kukas, much before they became part of the Indian national movement in the early twentieth century. • As the movement gained in popularity, the British took several steps to crush it in the period between 1863 and 1872.

www.insightsonindia.com 24 Insights IAS INSTA 75 Days REVISION PLAN for Prelims 2020 - InstaTests

29. Consider the following statements regarding Faraizi Movement 1. The Faraizis were the followers of a Muslim sect founded by Haji Shariat-Allah 2. It aimed at the eradication of social innovations or un-Islamic practices among the Muslims 3. The movement was merely a religious movement without political overtones. Which of the statements given above is/are correct? (a) 1 and 2 only (b) 2 and 3 only (c) 1 and 3 only (d) 1, 2 and 3

Solution: A

Faraizi Movement

• The Faraizis were the followers of a Muslim sect founded by Haji Shariat-Allah of Faridpur in Eastern Bengal. • The movement, also called the Fara’idi Movement because of its emphasis on the Islamic pillars of faith, was founded by Haji Shariatullah in 1818. • Its scene of action was East Bengal, and it aimed at the eradication of social innovations or un-Islamic practices current among the Muslims of the region and draw their attention to their duties as Muslims. • Under the leadership of Haji’s son, Dudu Mian, the movement became revolutionary from 1840 onwards. He gave the movement an organisational system from the village to the provincial level with a khalifa or authorised deputy at every level. • The Fara’idis organised a paramilitary force armed with clubs to fight the zamindars who were mostly Hindu, though there were some Muslim landlords too, besides the indigo planters. Dudu Mian asked his followers not to pay rent. The organisation even established its own Law courts. • Dudu Mian was arrested several times, and his arrest in 1847 finally weakened the movement. The movement survived merely as a religious movement without political overtones after the death of Dudu Mian in 1862.

30. Consider the following statements regarding Western Ghats 1. They are older than the Himalayas 2. It is one of the eight “hottest hot-spots” of biological diversity in the world 3. They are spanning over five states.

www.insightsonindia.com 25 Insights IAS INSTA 75 Days REVISION PLAN for Prelims 2020 - InstaTests

4. Kasturirangan Committee has proposed a Western Ghats Ecology Authority to regulate these activities in this area. Which of the statements given above is/are correct? (a) 1, 2 and 3 only (b) 1 and 2 only (c) 2 and 4 only (d) None of the above

Solution: A

• A public interest litigation petition has been filed in the Madras High Court seeking a direction to the Centre and State government to constitute a permanent body for taking serious steps to safeguard the flora, fauna and other natural resources in the Eastern and Western Ghat areas in Tamil Nadu. • The petition is on the basis of the recommendations made by the Madhav Gadgil and Kasturi Rangan committees. Importance of Western Ghats:

• The Western Ghats is an extensive region spanning over six States. It is the home of many endangered plants and animals. It is a UNESCO World Heritage site. • It is one of the eight “hottest hot-spots” of biological diversity in the world. • According to UNESCO, the Western Ghats are older than the Himalayas. They influence Indian monsoon weather patterns by intercepting the rain-laden monsoon winds that sweep in from the south-west during late summer. Gadgil Committee:

• It defined the boundaries of the Western Ghats for the purposes of ecological management. • It proposed that this entire area be designated as ecologically sensitive area (ESA). • Within this area, smaller regions were to be identified as ecologically sensitive zones (ESZ) I, II or III based on their existing condition and nature of threat. • It proposed to divide the area into about 2,200 grids, of which 75 per cent would fall under ESZ I or II or under already existing protected areas such as wildlife sanctuaries or natural parks. • The committee proposed a Western Ghats Ecology Authority to regulate these activities in the area.

www.insightsonindia.com 26 Insights IAS INSTA 75 Days REVISION PLAN for Prelims 2020 - InstaTests

www.insightsonindia.com 27 Insights IAS INSTA 75 Days REVISION PLAN for Prelims 2020 - InstaTests

Extra Reading: Kasturirangan Committee:

• None of the six concerned states agreed with the recommendations of the Gadgil Committee, which submitted its report in August 2011. • In August 2012, then Environment Minister constituted a High-Level Working Group on Western Ghats under Kasturirangan to “examine” the Gadgil Committee report in a “holistic and multidisciplinary fashion in the light of responses received” from states, central ministries and others. • The Kasturirangan report seeks to bring just 37% of the Western Ghats under the Ecologically Sensitive Area (ESA) zones — down from the 64% suggested by the Gadgil report. Recommendations of Kasturirangan Committee:

• A ban on mining, quarrying and sand mining. • No new thermal power projects, but hydro power projects allowed with restrictions. • A ban on new polluting industries. • Building and construction projects up to 20,000 sq m was to be allowed but townships were to be banned. • Forest diversion could be allowed with extra safeguards.

31. The ‘United for Biodiversity’ coalition has been launched by (a) Global Environment Facility (b) Conservation International (c) European Commission (d) World Wide Fund for Nature

Solution: C

• The European Commission (EC) has launched the ‘United for Biodiversity’ coalition. • It was launched on World Wildlife Day 2020- 3rd March. What is it?

• The coalition is made up of zoos, aquariums, botanical gardens, national parks, and natural history and science museums from around the world. • The coalition offers the opportunity for all such institutions to “join forces and boost public awareness about the nature crisis, ahead of the crucial COP-15 of the Convention on Biological Diversity in Kunming, China in October 2020.

www.insightsonindia.com 28 Insights IAS INSTA 75 Days REVISION PLAN for Prelims 2020 - InstaTests

A common pledge adopted:

• The coalition adopted a common pledge, citing the Intergovernmental Platform on Biodiversity and Ecosystem Services (IPBES) Global Assessment finding that one million species were already at risk of extinction, and appeals to visitors to each of their institutions to “raise their voice for nature.”

32. Consider the following statements regarding Prime Minister’s National Relief Fund (PMNRF) 1. It has been constituted by the Parliament 2. It is recognized as a Trust under the Indian Trusts Act, 1882 3. The contributions under the PMNRF can be qualified as corporate social responsibility Which of the statements given above is/are correct? (a) 3 only (b) 2 and 3 only (c) 1 and 3 only (d) 1, 2 and 3

Solution: A

Prime Minister’s National Relief Fund (PMNRF):

• In pursuance of an appeal by the then Prime Minister, Pt. Jawaharlal Nehru in January, 1948, the Prime Minister’s National Relief Fund (PMNRF) was established with public contributions to assist displaced persons from Pakistan. • The resources of the PMNRF are now utilized primarily to render immediate relief to families of those killed in natural calamities like floods, cyclones and earthquakes, etc. and to the victims of the major accidents and riots. • Assistance from PMNRF is also rendered, to partially defray the expenses for medical treatment like heart surgeries, kidney Key features:

• Disbursements are made with the approval of the Prime Minister. • PMNRF has not been constituted by the Parliament. • The fund is recognized as a Trust under the Income Tax Act and the same is managed by Prime Minister or multiple delegates for national causes. • PMNRF is exempt under Income Tax Act.

www.insightsonindia.com 29 Insights IAS INSTA 75 Days REVISION PLAN for Prelims 2020 - InstaTests

• Prime Minister is the Chairman of PMNRF and is assisted by Officers/ Staff on honorary basis. • These contributions also qualify as CSR (corporate social responsibility) spend for companies, making it more attractive in terms of tax exemptions.

33. Consider the following statements regarding The Santhal Rebellion 1. It was a rebellion against the zamindars later turned into an anti-British movement. 2. It was organized under the leadership of Buddho Bhagat. Which of the statements given above is/are correct? (a) 1 only (b) 2 only (c) Both 1 and 2 (d) Neither 1 nor 2

Solution: A

The Santhal Rebellion (1855-56)

• Continued oppression of the Santhals, an agricultural people, who had fled to settle in the plains of the Rajmahal hills (Bihar) led to the Santhal rebellion against the zamindars. • The money-lenders who had the support of the police among others had joined the zamindars to subject the peasants to oppressive exactions and dispossession of lands. • The rebellion turned into an anti-British movement. Under Sidhu and Kanhu, two brothers, the Santhals proclaimed an end to Company rule, and declared the area between Bhagalpur and Rajmahal as autonomous. The rebellion was suppressed by 1856. • The Kols rebellion in 1831 was under the leadership of Buddho Bhagat.

34. Consider the following statements regarding Raja Rammohan Roy 1. He had set up the Atmiya Sabha and Brahmo Sabha. 2. The long-term agenda of Samaj was to purify , to preach monotheism and to establish a new religion. 3. Raja Radhakant Deb organised the Dharma Sabha to propagate the ideals of Brahmo Samaj. Which of the statements given above is/are correct? www.insightsonindia.com 30 Insights IAS INSTA 75 Days REVISION PLAN for Prelims 2020 - InstaTests

(a) 1 only (b) 2 and 3 only (c) 1 and 2 only (d) 1, 2 and 3

Solution: A

Raja Rammohan Roy (1772-1833), often called the father of Indian Renaissance and the maker of Modern India, was a man of versatile genius.

• Rammohan Roy believed in the modern scientific approach and principles of human dignity and social equality. • He put his faith in monotheism. He wrote Gift to Monotheists (1809) and translated into Bengali the Vedas and the five Upanishads to prove his conviction that ancient Hindu texts support monotheism. • In 1814, he set up the Atmiya Sabha (or Society of Friends) in Calcutta to propagate the monotheistic ideals of the Vedanta and to campaign against idolatry, caste rigidities, meaningless rituals and other social ills. • Raja Rammohan Roy founded the Brahmo Sabha in August 1828; it was later renamed Brahmo Samaj. Through the Sabha he wanted to institutionalise his ideas and mission. • The long-term agenda of the Brahmo Samaj—to purify Hinduism and to preach monotheism—was based on the twin pillars of reason and the Vedas and Upanishads. • Rammohan Roy did not want to establish a new religion. He only wanted to purify Hinduism of the evil practices which had crept into it. Roy’s progressive ideas met with strong opposition from orthodox elements like Raja Radhakant Deb who organised the Dharma Sabha to counter Brahmo Samaj propaganda. Roy’s death in 1833 was a setback for the Samaj’s mission.

35. Consider the following statements regarding Balshastri Jambhekar 1. He established an orthodox society and stood for the preservation of the status quo in socio-religious matters. 2. He started the newspaper Darpan. Which of the statements given above is/are correct? (a) 1 only (b) 2 only (c) Both 1 and 2 (d) Neither 1 nor 2 www.insightsonindia.com 31 Insights IAS INSTA 75 Days REVISION PLAN for Prelims 2020 - InstaTests

Solution: B

Balshastri Jambhekar (1812-1846) was a pioneer of social reform through journalism in Bombay; he attacked Brahminical orthodoxy and tried to reform popular Hinduism. He started the newspaper Darpan in 1832.

• Known as the father of Marathi journalism, Jambhekar used the Darpan to awaken the people to awareness of social reforms, such as widow remarriage, and to instill in the masses a scientific approach to life. • In 1840, he started Digdarshan which published articles on scientific subjects as well as history. • Jambhekar founded the Bombay Native General Library and started the Native Improvement Society of which an offshoot was the Students Literary and Scientific Library. He was the first professor of Hindi at the Elphinston College, besides being a director of the Colaba Observatory.

36. Consider the following statements regarding The Theosophical Movement 1. Madame H.P. Blavatsky and Colonel M.S. Olcott founded the Theosophical Society in Adyar 2. The society sought to investigate the unexplained laws of nature and the powers latent in man. 3. It accepted the Hindu beliefs in reincarnation and karma and drew inspiration from the philosophy of the Upanishads. Which of the statements given above is/are correct? (a) 1 and 2 only (b) 2 and 3 only (c) 1 and 3 only (d) 1, 2 and 3

Solution: B

The Theosophical Movement

• A group of westerners led by Madame H.P. Blavatsky (1831-1891) and Colonel M.S. Olcott, who were inspired by Indian thought and culture, founded the Theosophical Society in New York City, United States in 1875.

www.insightsonindia.com 32 Insights IAS INSTA 75 Days REVISION PLAN for Prelims 2020 - InstaTests

• In 1882, they shifted their headquarters to Adyar, on the outskirts of Madras (at that time) in India. The society believed that a special relationship could be established between a person’s soul and God by contemplation, prayer, revelation, etc. • It accepted the Hindu beliefs in reincarnation and karma, and drew inspiration from the philosophy of the Upanishads and samkhya, yoga and Vedanta schools of thought. It aimed to work for universal brotherhood of humanity without distinction of race, creed, sex, caste or colour. • The society also sought to investigate the unexplained laws of nature and the powers latent in man. The Theosophical Movement came to be allied with the Hindu renaissance. It opposed child marriage and advocated the abolition of caste discrimination, uplift of outcastes, improvement in the condition of widows. • In India, the movement became somewhat popular with the election of Annie Besant (1847-1933) as its president after the death of Olcott in 1907. Annie Besant had come to India in 1893. • She laid the foundation of the Central Hindu College in Benaras in 1898 where both Hindu religion and Western scientific subjects were taught. The college became the nucleus for the formation of Benaras Hindu University in 1916. Annie Besant also did much for the cause of the education of women.

37. Consider the following statements regarding Collegium system 1. It is the system of appointment and transfer of judges that has evolved through multiple amendments to the constitution. 2. Judges of the higher judiciary are appointed only through the collegium system and the government has a role only after names have been decided by the collegium. Which of the statements given above is/are correct? (a) 1 only (b) 2 only (c) Both 1 and 2 (d) Neither 1 nor 2

Solution: B

• It is the system of appointment and transfer of judges that has evolved through judgments of the Supreme Court. It is not by an Act of Parliament or by a provision of the Constitution. • The Supreme Court collegium is headed by the Chief Justice of India and comprises four other senior most judges of the court. The collegium system has its genesis in a www.insightsonindia.com 33 Insights IAS INSTA 75 Days REVISION PLAN for Prelims 2020 - InstaTests

series of judgments through interpretations of pertinent constitutional provisions called “Judges Cases”. Government’s Role

• Judges of the higher judiciary are appointed only through the collegium system and the government has a role only after names have been decided by the collegium. • It can also raise objections and seek clarifications regarding the collegium’s choices, but if the collegium reiterates the same names, the government is bound, under Constitution Bench judgments, to appoint them as judges

38. Consider the following statements 1. Election commission can take any action it deems fit to ensure that elections and the election process are free and fair. 2. Model code of conduct does not have statutory value and it is enforced by the moral and constitutional authority of the Election commission. Which of the statements given above is/are correct? (a) 1 only (b) 2 only (c) Both 1 and 2 (d) Neither 1 nor 2

Solution: C

• The Election Commission of India is directly established by Article 324 of the Constitution. It vests in the Commission the superintendence, direction and control of all elections to Parliament, the State legislatures, and the offices of the President and Vice-President. • The EC can take any action it deems fit to ensure that elections and the election process are free and fair. • The Model Code of Conduct is a set of guidelines issued by the Election Commission (EC) to regulate political parties and candidates for the conduct of free and fair elections. The code comes into force on the announcement of the poll schedule and remains operational till the process is concluded. • It does not have statutory value, and it is enforced only by the moral and constitutional authority of the EC. It is normally legally enforced by invoking other statutes such as the Indian Penal Code, 1860, Code of Criminal Procedure, 1973, and Representation of the People Act, 1951.

www.insightsonindia.com 34 Insights IAS INSTA 75 Days REVISION PLAN for Prelims 2020 - InstaTests

39. Consider the following statements regarding Deccan Riots 1. The ryots of Deccan region of western India suffered heavy taxation under the Ryotwari system. 2. A social boycott movement organized by the ryots against the outsider moneylenders. 3. The Deccan Agriculturists Relief Act was passed by Government as a conciliatory measure. Which of the statements given above is/are correct? (a) 1 and 2 only (b) 2 and 3 only (c) 1 and 3 only (d) 1, 2 and 3

Solution: D

• The ryots of Deccan region of western India suffered heavy taxation under the Ryotwari system. • Here again the peasants found themselves trapped in a vicious network with the moneylender as the exploiter and the main beneficiary. These moneylenders were mostly outsiders—Marwaris or Gujaratis. • The conditions had worsened due to a crash in cotton prices after the end of the American Civil War in 1864, the Government’s decision to raise the land revenue by 50% in 1867, and a succession of bad harvests. • In 1874, the growing tension between the moneylenders and the peasants resulted in a social boycott movement organized by the ryots against the “outsider” moneylenders. • The ryots refused to buy from their shops. No peasant would cultivate their fields. The barbers, washermen, shoemakers would not serve them. This social boycott spread rapidly to the villages of Poona, Ahmednagar, Sholapur and Satara. Soon the social boycott was transformed into agrarian riots with systematic attacks on the moneylenders’ houses and shops. • The debt bonds and deeds were seized and publicly burnt. The Government succeeded in repressing the movement. • As a conciliatory measure, the Deccan Agriculturists Relief Act was passed in 1879. This time also, the modern nationalist intelligentsia of Maharashtra supported the peasants’ cause.

www.insightsonindia.com 35 Insights IAS INSTA 75 Days REVISION PLAN for Prelims 2020 - InstaTests

40. Which of the following vows were taken under Eka or the Unity Movement 1. Pay only the recorded rent but would pay it on time 2. Give no help to criminals 3. Refuse to do forced labour 4. Abide by panchayat decisions Which of the statements given above is/are correct? (a) 1, 2 and 3 only (b) 2, 3 and 4 only (c) 1, 3 and 4 only (d) 1, 2, 3 and 4

Solution: D

Eka Movement Towards the end of 1921, peasant discontent resurfaced in some northern districts of the United Provinces—Hardoi, Bahraich, Sitapur. The issues involved were: 1. high rents—50 per cent higher than the recorded rates; 2. oppression of thikadars in charge of revenue collection; and 3. practice of share-rents. The meetings of the Eka or the Unity Movement involved a symbolic religious ritual in which the assembled peasants vowed that they would 1. pay only the recorded rent but would pay it on time; 2. not leave when evicted; 3. refuse to do forced labour; 4. give no help to criminals; 5. abide by panchayat decisions. The grassroot leadership of the Eka Movement came from Madari Pasi and other low-caste leaders, and many small zamindars. By March 1922, severe repression by authorities brought the movement to an end.

41. Consider the following statements regarding Petroleum & Explosives Safety Organization 1. It functions under the Ministry of Petroleum and Natural Gas

www.insightsonindia.com 36 Insights IAS INSTA 75 Days REVISION PLAN for Prelims 2020 - InstaTests

2. It is the apex department to control and administer manufacture, storage, transport and handling of explosives, petroleum, compressed gases and other hazardous substances in India. Which of the statements given above is/are correct? (a) 1 only (b) 2 only (c) Both 1 and 2 (d) Neither 1 nor 2

Solution: B

Petroleum & Explosives Safety Organization:

• It is a department under Department for the Promotion of Industry and Internal Trade under Ministry of Commerce and Industry. • It is a regulatory authority with autonomous status. • It was established during the British India in 1890s as Department of Explosives and later expanded to various other activities. • As a statutory authority, PESO is entrusted with the responsibilities under the Explosives Act, 1884; Petroleum Act, 1934; Inflammable Substances Act, 1952, Environment (Protection Act), 1986. • Why in News? Petroleum & Explosives Safety Organization takes various measures to address the problems faced by Petroleum, Explosives, Oxygen and Industrial Gas Industries.

42. Consider the following statements regarding Kyasanoor Forest Disease (KFD) 1. It is endemic to the Karnataka state. 2. It can transmit through person’s saliva and body fluid. 3. There is no vaccine for KFD. Which of the statements given above is/are correct? (a) 1 and 2 only (b) 1 only (c) 1 and 3 only (d) 1, 2 and 3

www.insightsonindia.com 37 Insights IAS INSTA 75 Days REVISION PLAN for Prelims 2020 - InstaTests

Solution: B

Kyasanur Forest Disease

• KFD is caused by the Kyasanur Forest Disease Virus (KFDV). The virus was identified in 1957 when it was isolated from a sick monkey from the Kyasanur Forest. Since then, between 400-500 humans cases per year have been reported. • Hard ticks (Hemaphysalis spinigera) are the reservoir of the KFD virus and once infected, remain so for life. • Rodents, shrews, and monkeys are common hosts for KFDV after being bitten by an infected tick. KFDV can cause epizootics with high fatality in primates. • KFD is endemic to the Indian state of Karnataka. • The virus is transmitted to human beings through parasitic ticks which latch on to monkeys. No person-to-person transmission has been described. • A vaccine does exist for KFD and is used in endemic areas of India.

43. Consider the following statements regarding All India Trade Union Congress 1. The All India Trade Union Congress was founded in 1920 by Lala Lajpat Rai. 2. The AITUC was influenced by social democratic ideas of the British Labour Party and Gandhian philosophy. Which of the statements given above is/are correct? (a) 1 only (b) 2 only (c) Both 1 and 2 (d) Neither 1 nor 2

Solution: B

• A follower of Gokhale, Narayan Malhar Joshi founded the Social Service League in Bombay with an aim to secure for the masses better and reasonable conditions of life and work. Joshi also founded the All India Trade Union Congress (1920). • The All India Trade Union Congress was founded on October 31, 1920. The Indian National Congress president for the year, Lala Lajpat Rai, was elected as the first president of AITUC and Dewan Chaman Lal as the first general secretary. • Lajpat Rai was the first to link capitalism with imperialism—“imperialism and militarism are the twin children of capitalism”.

www.insightsonindia.com 38 Insights IAS INSTA 75 Days REVISION PLAN for Prelims 2020 - InstaTests

• The prominent Congress and swarajist leader C.R. Das presided over the third and the fourth sessions of the AITUC. • The Gaya session of the Congress (1922) welcomed the formation of the AITUC and a committee was formed to assist it. C.R. Das advocated that the Congress should take up the workers’ and peasants’ cause and incorporate them in the struggle for swaraj or else they would get isolated from the movement. Other leaders who kept close contacts with the AITUC included Nehru, Subhas Bose, C.F. Andrews, J.M. Sengupta, Satyamurthy, V.V. Giri and Sarojini Naidu. In the beginning, the AITUC was influenced by social democratic ideas of the British Labour Party. The Gandhian philosophy of non-violence, trusteeship and class-collaboration had great influence on the movement. Gandhi helped organize the Ahmedabad Textile Labour Association (1918) and through a protest secured a 27.5 per cent wage hike.

44. Which of the following are the features of The Trade Union Act, 1926 1. Recognized trade unions as legal associations 2. Secured immunity, both civil and criminal, for trade unions from prosecution for legitimate activities 3. It made the strikes in public utility services illegal. Which of the statements given above is/are correct? (a) 1 and 2 only (b) 2 and 3 only (c) 1 and 3 only (d) 1, 2 and 3

Solution: A

The Trade Union Act, 1926

• recognised trade unions as legal associations; • laid down conditions for registration and regulation of trade union activities; • secured immunity, both civil and criminal, for trade unions from prosecution for legitimate activities, but put some restrictions on their political activities. The Trade Disputes Act (TDA), 1929

• made compulsory the appointment of Courts of Inquiry and Consultation Boards for settling industrial disputes;

www.insightsonindia.com 39 Insights IAS INSTA 75 Days REVISION PLAN for Prelims 2020 - InstaTests

• made illegal the strikes in public utility services like posts, railways, water and electricity, unless each individual worker planning to go on strike gave an advance notice of one month to the administration; • forbade trade union activity of coercive or purely political nature and even sympathetic strikes.

45. Nair movement was led by (a) Shri Narayana Guru (b) Dr T.M. Nair, P. Chetti and C.N. Mudalair (c) E.V. Ramaswami Naiker (d) C.V. Raman Pillai, K. Rama Pillai, and M. Padmanabha Pillai

Solution: D

• Aravippuram Movement led by Shri Narayana Guru • Justice party Movement led by Dr T.M. Nair, P. Chetti and C.N. Mudalair on behalf of castes • Nair Movement led by C.V. Raman Pillai, K. Rama Pillai, and M. Padmanabha Pillai • Self-respect Movement was led by E.V. Ramaswami Naiker

46. Consider the following statements regarding Appropriation Bill 1. The Rajya Sabha doesn’t have the power to recommend any amendments in this Bill. 2. Once the Appropriation Bill has been passed, only President can propose amendments. Which of the statements given above is/are correct? (a) 1 only (b) 2 only (c) Both 1 and 2 (d) Neither 1 nor 2

Solution: D

www.insightsonindia.com 40 Insights IAS INSTA 75 Days REVISION PLAN for Prelims 2020 - InstaTests

Appropriation Bill:

• Appropriation Bill is a money bill that allows the government to withdraw funds from the Consolidated Fund of India to meet its expenses during the course of a financial year. • As per article 114 of the Constitution, the government can withdraw money from the Consolidated Fund only after receiving approval from Parliament. • To put it simply, the Finance Bill contains provisions on financing the expenditure of the government, and Appropriation Bill specifies the quantum and purpose for withdrawing money. Procedure followed:

• The government introduces the Appropriation Bill in the lower house of Parliament after discussions on Budget proposals and Voting on Demand for Grants. • The Appropriation Bill is first passed by the Lok Sabha and then sent to the Rajya Sabha. • The Rajya Sabha has the power to recommend any amendments in this Bill. However, it is the prerogative of the Lok Sabha to either accept or reject the recommendations made by the upper house of Parliament. • The unique feature of the Appropriation Bill is its automatic repeal clause, whereby the Act gets repealed by itself after it meets its statutory purpose. • Once the Appropriation Bill has been passed, no amendments in its amounts can be proposed in either Parliament or legislature.

47. Consider the following statements regarding The Pagal Panthis 1. They are semi-religious group mainly constituting the Hajong and Garo tribes. 2. They refused to pay rent above a certain limit and attacked the houses of zamindars. Which of the statements given above is/are correct? (a) 1 only (b) 2 only (c) Both 1 and 2 (d) Neither 1 nor 2

Solution: C

www.insightsonindia.com 41 Insights IAS INSTA 75 Days REVISION PLAN for Prelims 2020 - InstaTests

The Pagal Panthis

• The Pagal Panthi, a semi-religious group mainly constituting the Hajong and Garo tribes of Mymensingh district (earlier in Bengal), was founded by Karam Shah. But the tribal peasants organised themselves under Karam Shah’s son, Tipu, to fight the oppression of the zamindars. • From 1825 to 1835, the Pagal Panthis refused to pay rent above a certain limit and attacked the houses of zamindars. The government introduced an equitable arrangement to protect these peasants, but the movement was violently suppressed.

48. Consider the following statements regarding The Servants of India Society 1. Gopal Krishna Gokhale founded the Servants of India Society. 2. The aim of the society was to train national missionaries for the service of India 3. The Hitavada was published to project the views of the society. Which of the statements given above is/are correct? (a) 1 and 2 only (b) 2 and 3 only (c) 1 and 3 only (d) 1, 2 and 3

Solution: D

The Servants of India Society

• Gopal Krishna Gokhale (1866-1915), a liberal leader of the Indian National Congress, founded the Servants of India Society in 1905 with the help of M.G. Ranade. • The aim of the society was to train national missionaries for the service of India; to promote, by all constitutional means, the true interests of the Indian people; and to prepare a cadre of selfless workers who were to devote their lives to the cause of the country in a religious spirit. • In 1911, the Hitavada began to be published to project the views of the society. • The society chose to remain aloof from political activities and organizations like the Indian National Congress. • After Gokhale’s death (1915), Srinivasa Shastri took over as president. The society still continues to function, though with a shrunken base, at many places in India. It works in the field of education, providing ashram type of schools for tribal girls and balwadis at many places.

www.insightsonindia.com 42 Insights IAS INSTA 75 Days REVISION PLAN for Prelims 2020 - InstaTests

49. Consider the following statements regarding International Maritime Organisation (IMO) 1. It is a United Nations specialized agency with responsibility for the safety and security of shipping and the prevention of marine pollution by ships. 2. India has not ratified the treaty. Which of the statements given above is/are correct? (a) 1 only (b) 2 only (c) Both 1 and 2 (d) Neither 1 nor 2

Solution: A

• International Maritime Organisation (IMO) is a United Nations specialized agency with responsibility for the safety and security of shipping and the prevention of marine pollution by ships. • Its main role is to create a regulatory framework for the shipping industry that is fair and effective, universally adopted and universally implemented. • India has been one of the earliest members of the IMO, having ratified its Convention and joined it as a member-state in the year 1959. • India has had the privilege of being elected to and serving the Council of the IMO, ever since it started functioning, and till date, except for two years for the period 1983- 1984. • India is a party to 34 IMO Conventions and protocols and is currently in the advanced stage of ratifying Ballast Water Convention and Bunker Convention. Extra Reading IMO Council

• The IMO Council consists of 40 member countries. • In Categories A” and B” there are 10 members each and in Category C” 20 members, who are elected by the IMO Assembly. • IMO Council plays a crucial role to play in deciding various important matters within the mandate of the IMO, in relation to the global shipping industry, including its work programme strategy and budget. • Ballast Water Convention and Bunker Convention.

www.insightsonindia.com 43 Insights IAS INSTA 75 Days REVISION PLAN for Prelims 2020 - InstaTests

50. Consider the following statements regarding Event Horizon Telescope (EHT) 1. EHT project was devised by NASA in 2012 to directly observe the immediate environment of a black hole. 2. It is a network of 10 radio telescopes on four continents that collectively operate like a single instrument nearly the size of the Earth. Which of the statements given above is/are correct? (a) 1 only (b) 2 only (c) Both 1 and 2 (d) Neither 1 nor 2

Solution: B

• The EHT is a project of network of 10 radio telescopes on four continents that collectively operate like a single instrument nearly the size of the Earth. • The EHT project, an international partnership formed in 2012 is to directly observe the immediate environment of a black hole. • Its aim was to capture the first image of a black hole by creating a virtual Earth-sized telescope. • It is a project to create a large telescope array consisting of a global network of radio telescopes and combining data from several verylong-baseline interferometry (VLBI) stations around the Earth • This technique of linking radio dishes across the globe to create an Earth-sized interferometer has been used to measure the size of the emission regions of the two supermassive black holes. https://eventhorizontelescope.org/about

DAY – 35 (InstaTest-35)

51. Consider the following statements 1. Sagittarius A* is believed to be a super massive galaxy present in the Universe. 2. The Kuiper belt a circumstellar disc in the outer Solar System, extending from the orbit of Neptune to approximately 50 AU from the Sun. Which of the statements given above is/are correct? (a) 1 only

www.insightsonindia.com 44 Insights IAS INSTA 75 Days REVISION PLAN for Prelims 2020 - InstaTests

(b) 2 only (c) Both 1 and 2 (d) Neither 1 nor 2

Solution: B

• Sagittarius A or Sgr A is a complex radio source at the center of the Milky Way which contains a supermassive black hole. It is located in the constellation Sagittarius, and is hidden from view at optical wavelengths by large clouds of cosmic dust in the spiral arms of the Milky Way. • Kuiper Belt — a donut-shaped region of icy bodies beyond the orbit of Neptune. There may be millions of these icy objects, collectively referred to as Kuiper Belt objects (KBOs) or trans-Neptunian objects (TNOs), in this distant region of our solar system. It is extending from the orbit of Neptune to approximately 50 AU from the Sun.

https://www.thehindu.com/sci-tech/science/watch-all-about-black- holes/article29446293.ece

52. Consider the following statements 1. Treaty of Sagauli was signed following First Burma War. 2. Treaty of Yandabo was signed between British and Gorkhas. www.insightsonindia.com 45 Insights IAS INSTA 75 Days REVISION PLAN for Prelims 2020 - InstaTests

Which of the statements given above is/are correct? (a) 1 only (b) 2 only (c) Both 1 and 2 (d) Neither 1 nor 2

Solution: D

Anglo-Nepalese Relations

• The Gorkhas wrested control of Nepal from the successors of Ranjit Malla of Bhatgaon in 1760. They began to expand their dominion beyond the mountains. They found it easier to expand in the southern direction, as the north was well defended by the Chinese. • In 1801, the English annexed Gorakhpur which brought the Gorkhas’ boundary and the Company’s boundary together. The conflict started due to the Gorkhas’ capture of Butwal and Sheoraj in the period of Lord Hastings (1813-23). The war, ended in the Treaty of Sagauli, 1816 which was in favour of the British. First Burma War (1824-26)

• The first war with Burma was fought when the Burmese expansion westwards and occupation of Arakan and Manipur, and the threat to Assam and the Brahmaputra Valley led to continuous friction along the ill-defined border between Bengal and Burma, in the opening decades of the nineteenth century. The British expeditionary forces occupied Rangoon in May 1824 and reached within 72 km of the capital at Ava. Peace was established in 1826 with the Treaty of Yandabo.

53. Consider the following pairs regarding foreign policy with Afghanistan 1. Forward Policy : Auckland 2. Policy of Masterly Inactivity : John Lawrence 3. Policy of Proud Reserve : Lytton Which of the pairs given above is/are matched correctly? (a) 3 only (b) 1 and 2 only (c) 2 and 3 only (d) 1, 2 and 3

www.insightsonindia.com 46 Insights IAS INSTA 75 Days REVISION PLAN for Prelims 2020 - InstaTests

Solution: D

Forward Policy of Auckland

• Auckland who came to India as the governor-general in 1836, advocated a forward policy. This implied that the Company government in India itself had to take initiatives to protect the boundary of British India from a probable Russian attack. • This objective was to be achieved either through treaties with the neighbouring countries or by annexing them completely. John Lawrence and the Policy of Masterly Inactivity

• John Lawrence (1864-1869) started a policy of masterly inactivity which was a reaction to the disasters of the First Afghan War and an outcome of practical common sense and an intimate knowledge of the frontier problem and of Afghan passion for independence. Lytton and the Policy of Proud Reserve

• Lytton, a nominee of the Conservative government under Benjamin Disraeli (1874- 80), became the Viceroy of India in 1876. He started a new foreign policy of ‘proud reserve’, which was aimed at having scientific frontiers and safeguarding ‘spheres of influence’. According to Lytton, the relations with Afghanistan could no longer be left ambiguous.

54. Which of the following rights were guaranteed under Karachi Resolution on Fundamental Rights 1. Free speech and free press 2. Free and compulsory primary education 3. Universal adult franchise 4. Equal legal rights irrespective of caste, creed and sex Select the correct answer using the code given below (a) 1, 2 and 3 only (b) 2, 3 and 4 only (c) 1, 3 and 4 only (d) 1, 2, 3 and 4

Solution: D

www.insightsonindia.com 47 Insights IAS INSTA 75 Days REVISION PLAN for Prelims 2020 - InstaTests

Karachi Congress Session—1931

• In March 1931, a special session of the Congress was held at Karachi to endorse the Gandhi-Irwin Pact. Six days before the session (which was held on March 29) Bhagat Singh, Sukhdev and Rajguru were executed. Throughout Gandhi’s route to Karachi, he was greeted with black flag demonstrations by the Punjab Naujawan Bharat Sabha, in protest against his failure to secure commutation of the death sentence for Bhagat and his comrades. Congress Resolutions at Karachi

• While disapproving of and dissociating itself from political violence, the Congress admired the ‘bravery’ and ‘sacrifice’ of the three martyrs. • The Delhi Pact or Gandhi-Irwin Pact was endorsed. • The goal of purna swaraj was reiterated. • Two resolutions were adopted—one on Fundamental Rights and the other on National Economic Programme which made the session particularly memorable. The Resolution on Fundamental Rights guaranteed— 1. free speech and free press 2. right to form associations 3. right to assemble 4. universal adult franchise 5. equal legal rights irrespective of caste, creed and sex 6. neutrality of state in religious matters 7. free and compulsory primary education 8. protection to culture, language, script of minorities and linguistic groups

55. Consider the following statements regarding Haripura session 1. This was the session held in a village for the first time. 2. Subhash Chandra Bose was unanimously elected president of the session. 3. National Planning Committee was set up under the chairmanship of Jawaharlal Nehru. Which of the statements given above is/are correct? (a) 1 and 2 only (b) 2 and 3 only (c) 1 and 3 only (d) 1, 2 and 3

Solution: B

www.insightsonindia.com 48 Insights IAS INSTA 75 Days REVISION PLAN for Prelims 2020 - InstaTests

Haripura

• At the Congress meeting in Haripura, Gujarat, in February 1938, Bose was unanimously elected president of the session. He was firm in his belief that the Congress ministries in the provinces had immense revolutionary potential, as he said in his presidential address. Bose also talked of economic development of the country through planning and was instrumental in setting up a National Planning Committee later. • The session adopted a resolution that the Congress would give moral support to those who were agitating against the governance in the princely states. • In the following months, the international situation was highly disturbed; there were clear signs that Europe was going to be embroiled in war. • National Planning Committee was set up under the chairmanship of Jawaharlal Nehru. • Faizpur 1936: was the session held in a village for the first time.

56. Consider the following statements regarding Nilgiri Tahr 1. It is endemic to the Nilgiri Hills and Western Ghats 2. It has been listed as Critically Endangered by IUCN. 3. It is found in open montane grassland habitat of rain forests eco region. Which of the statements given above is/are correct? (a) 1 and 2 only (b) 2 and 3 only (c) 1 and 3 only (d) 1, 2 and 3

Solution: C

Nilgiri Tahr:

• Recently, Nilgiri tahr’s population has increased from 568 in 2018 to 612 in 2019 in the Mukurthi National Park, Tamil Nadu. • Nilgiri Tahr is also known as Nilgiri Ibex. • State animal of Tamil Nadu • Only species of Caprine ungulate that is found south of the Himalayas in India • Habitat: Open montane grassland habitat of the South Western Ghats montane rain forests Eco region. Endemic to the Nilgiri Hills and the southern portion of the Western Ghats in the states of Tamil Nadu and Kerala in Southern India (ex: Anamalai Hills, Palni Hills) www.insightsonindia.com 49 Insights IAS INSTA 75 Days REVISION PLAN for Prelims 2020 - InstaTests

• It has been listed as “Endangered” by IUCN. • It has been listed under Schedule 1 of the Wildlife (Protection) Act, 1972 which provides absolute protection and offences under these are prescribed the highest penalties • The Adult males of Nilgiri Tahr species develop a light grey area or “saddle” on their backs and are hence called “Saddlebacks”

57. Consider the following statements regarding civil services reforms 1. Cornwallis was the first to bring into existence and organize the civil services. 2. Aitchison Committee decreased the age limit to 21. Which of the statements given above is/are correct? (a) 1 only (b) 2 only (c) Both 1 and 2 (d) Neither 1 nor 2

Solution: A

www.insightsonindia.com 50 Insights IAS INSTA 75 Days REVISION PLAN for Prelims 2020 - InstaTests

Cornwallis (governor-general, 1786-93) was the first to bring into existence and organize the civil services. He tried to check corruption through—

• raising the civil servants’ salary, • strict enforcement of rules against private trade, • debarring civil servants from taking presents, bribes etc., • enforcing promotions through seniority. The Aitchison Committee on Public Services (1886), set up by Dufferin, recommended—

• dropping of the terms ‘covenanted’ and ‘uncovenanted’; • classification of the civil service into Imperial Indian Civil Service (examination in England), Provincial Civil Service (examination in India) and Subordinate Civil Service (examination in India); and, • raising the age limit to 23.

58. Match the following committees with the purpose it was setup for: 1. Hilton Young Commission Labour Reforms 2. Mac Donnell Commission Police Reforms 3. Floud Commission Bengal Land Revenue 4. Butler Commission Indian States relation with British Crown Which of the pairs given above is/are matched correctly? (a) 1 and 2 only (b) 2 and 3 only (c) 3 and 4 only (d) 1 and 4 only

Solution: C

1. The Hilton Young Commission was a Commission of Inquiry appointed in 1926 to look into the possible closer union of the British territories in East and Central Africa. These were individually economically underdeveloped, and it was suggested that some form of association would result both in cost savings and their more rapid development. 2. The MacDonnell Commission was the famine commission appointed by Lord Curzon, after a long period of 20 years, when Lord Lytton had tried to formulate the general principles of dealing with such famines. 3. Floud Commission a land revenue commission established by the government of Bengal in 1938 4. The Indian states committee appointed a committee under the Chairmanship of Sir Harcourt Butler which was popularly known as ‘the Butler Committee’ to investigate www.insightsonindia.com 51 Insights IAS INSTA 75 Days REVISION PLAN for Prelims 2020 - InstaTests

and clarify the relationship between the paramount power and the Princes of Princely States in AD 1927.

Extra Reading:

Famine Commissions

1866 : Campbell Commission Viceroy : John Lawrence

1880 : Stratchy Commission (Famine Commission) Viceroy : Lytton Chairman : Richard Strachey Objective : Give relief famine sricken

1897 : Lyall Commission (Famine Commission) Viceroy : Elgin Chairman : James Lyall Objective : Suggestion to earlier report

1901 : Mac Donnell Commission (Famine Commission) Viceroy : Curzon Chairman : Anthony MacDonnel Objective : Investigate events of Bengal famine

Currency Commissions 1886 : Mansfield Commission by Dufferin 1898 : Fowler Commission by Elgin II 1919 : Babington Smith Commission by Chelmsford 1926 : Hilton Young Commission by Linlithgow www.insightsonindia.com 52 Insights IAS INSTA 75 Days REVISION PLAN for Prelims 2020 - InstaTests

Other Commissions

1901 : Scott-Moncrieff Commission (Irrigation) by Curzon 1902 : Fraser Commission (Police Reforms) by Curzon 1919 : Hunter Commission (Punjab Disturbances) by Chelmsford 1927 : Butler Commission (Indian States relation with British Crown) by Irwin 1929 : Whitelay Commission (Labour) by Irwin 1935 : Sapru Commission (Unemployment) by Linlithgow 1939 : Chalfield Commission (Army) by Linlighgow 1940 : Floud Commission (Bengal Land Revenue) by Linlighgow

59. Consider the following statements regarding Local self-government 1. Financial decentralization was a legislative devolution inaugurated by the Indian Councils Act of 1861. 2. Lord Mayo is called father of local self-government in India. Which of the statements given above is/are correct? (a) 1 only (b) 2 only (c) Both 1 and 2 (d) Neither 1 nor 2

Solution: A

Mayo’s Resolution of 1870

• Financial decentralization was a legislative devolution inaugurated by the Indian Councils Act of 1861. Apart from the annual grant from imperial Government, the provincial governments were authorized to resort to local taxation to balance their budgets. This was done in context of transfer of certain departments of administration, such as medical services, education and roads, to the control of provincial governments. This was the beginning of local finance. Mayo’s Resolution emphasized, “Local interest, supervision and care are necessary for success in the management of the funds devoted to education, sanitation, medical relief and local public works.”

www.insightsonindia.com 53 Insights IAS INSTA 75 Days REVISION PLAN for Prelims 2020 - InstaTests

Ripon’s Resolution of 1882

• The Government of Ripon desired the provincial governments to apply in case of local bodies the same principle of financial decentralization which Lord Mayo’s Government had begun towards them. For his contributions, Lord Ripon is called father of local self-government in India.

60. Consider the following statements regarding Mt Aconcagua 1. It is located in Uruguay 2. It is part of the Andes mountain range Which of the statements given above is/are correct? (a) 1 only (b) 2 only (c) Both 1 and 2 (d) Neither 1 nor 2

Solution: B

www.insightsonindia.com 54 Insights IAS INSTA 75 Days REVISION PLAN for Prelims 2020 - InstaTests

• A 12-year old Mumbai student, Kaamya Karthikeyan has set a record of becoming the youngest in the world to summit Mt. Aconcagua. About Mt. Aconcagua:

• It is the highest peak of the Andes Mountains in Argentina, South America. • It is the highest mountain outside of Asia, with a summit elevation of 6,960.8 metres. • The mountain is one of the so-called Seven Summits of the seven continents. • It is Part of the Andes mountain range in South America.

61. Consider the following statements regarding National Organic food Festival 1. It will be jointly organized by Ministry of Food Processing Industries and Ministry of Women and Child Development. 2. It aims to strengthen the organic market and empower women entrepreneurs in the area of production and processing of organic products. Which of the statements given above is/are correct? (a) 1 only (b) 2 only (c) Both 1 and 2 (d) Neither 1 nor 2

Solution: C

National Organic food Festival

• It will be jointly organized by Ministry of Food Processing Industries (MoFPI) and Ministry of Women and Child Development (Mo WCD). • Theme of the festival is Unleashing India’s Organic Market Potential. • The event will focus on facilitating business linkages and empowering women entrepreneurs through pre-arranged B2B and B2G meetings. • It aims to strengthen the organic market and empower women entrepreneurs in the area of production and processing of organic products. • Women Entrepreneurs and Self Help groups (SHG’s) from all over the country will be exhibiting their organic products in various segments such as fruit & vegetables, ready to eat products, spices and condiments, honey, cereals, dry fruits etc.

www.insightsonindia.com 55 Insights IAS INSTA 75 Days REVISION PLAN for Prelims 2020 - InstaTests

62. Which of the following was/were the provisions of Indian Factory Act, 1891 1. Employment of children under 7 years of age prohibited 2. It fixed maximum working hours for women at 11 hours per day. 3. It provided weekly holiday for all. Select the correct answer using the code given below (a) 1 only (b) 2 and 3 only (c) 1 and 2 only (d) 1, 2 and 3

Solution: B

The Indian Factory Act, 1881 dealt primarily with the problem of child labour (between 7 and 12 years of age). Its significant provisions were:

• employment of children under 7 years of age prohibited, • working hours restricted to 9 hours per day for children, • children to get four holidays in a month, • hazardous machinery to be properly fenced off. The Indian Factory Act, 1891 1. increased the minimum age (from 7 to 9 years) and the maximum (from 12 to 14 years) for children, 2. reduced maximum working hours for children to 7 hours a day, 3. fixed maximum working hours for women at 11 hours per day with an one-and-a-half hour interval (working hours for men were left unregulated), 4. provided weekly holiday for all.

63. Consider the following statements regarding Siddi Tribes 1. They are originated from Africa. 2. They are largely living in Karnataka, Andhra Pradesh, Gujarat and Maharashtra states. 3. They are included in the Particularly Vulnerable Tribal Groups. Which of the statements given above is/are correct? (a) 1 and 3 only

www.insightsonindia.com 56 Insights IAS INSTA 75 Days REVISION PLAN for Prelims 2020 - InstaTests

(b) 2 only (c) 3 only (d) 1, 2 and 3

Solution: A

• The Siddi community is also known by different synonyms such as Habshi and Badsha. It is believed that they are of African origin because they clearly show the Negroid racial strain in their physical features. Descendants of Bantu people of East Africa, Siddi ancestors were largely brought to India as slaves by Arabs as early as the 7th Century, followed by the Portuguese and the British later on. • At present, the Siddis are living on the western coast of Gujarat, Maharashtra and Karnataka states. In Karnataka, they mainly live in Dharwad, Belagavi and Uttar Kannada districts. • In India, the Union government in 2003, classified Siddis under the list of Scheduled Tribes. They are included in the Centre’s list of Particularly Vulnerable Tribal Groups

64. Who among the following was/were Economic critic of British rule? 1. Dadabhai Naoroji 2. Romesh Chandra Dutt 3. Gopal Krishna Gokhale 4. Prithwishchandra Ray Select the correct answer using the code given below (a) 1, 2 and 3 only (b) 2, 3 and 4 only (c) 1, 3 and 4 only (d) 1, 2, 3 and 4

Solution: D

• Dadabhai Naoroji, the ‘Grand Old Man of India’, who after a brilliant analysis of the colonial economy put forward the theory of economic drain in Poverty and UnBritish Rule in India.

www.insightsonindia.com 57 Insights IAS INSTA 75 Days REVISION PLAN for Prelims 2020 - InstaTests

• Other economic analysts included Justice Mahadeo Govind Ranade, Romesh Chandra Dutt (The Economic History of India), Gopal Krishna Gokhale, G. Subramaniya Iyer and Prithwishchandra Ray. • The essence of nineteenth century colonialism, they said, lay in the transformation of India into a supplier of foodstuffs and raw-materials to the metropolis, a market for metropolitan manufacturers and a field for investment of British capital. • These early nationalist analysts organised intellectual agitations and advocated a complete severance of India’s economic subservience to Britain and the development of an independent economy based on modern industries.

65. Consider the following statements regarding Press reforms 1. Indian Press Act, 1910 reversed the worst features of the Vernacular Press Act. 2. Indian Press Act, 1910 is popularly known as Metcalfe Act. Which of the statements given above is/are correct? (a) 1 only (b) 2 only (c) Both 1 and 2 (d) Neither 1 nor 2

Solution: D

• In 1883, Surendranath Banerjea became the first Indian journalist to be imprisoned. In an angry editorial in The Bengalee Banerjea had criticised a judge of Calcutta High Court for being insensitive to the religious sentiments of Bengalis in one of his judgements. • Bal Gangadhar Tilak is most frequently associated with the nationalist fight for the freedom of press. • Indian Press Act, 1910: This Act revived the worst features of the VPA—local government was empowered to demand a security at registration from the printer/publisher and fortfeit/deregister if it was an offending newspaper, and the printer of a newspaper was required to submit two copies of each issue to local government free of charge. Press Act of 1835 or Metcalfe Act

• Metcalfe (governor-general—1835-36) repealed the obnoxious 1823 ordinance and earned the epithet, “liberator of the Indian press”. The new Press Act (1835) required a printer/publisher to give a precise account of premises of a publication and cease functioning, if required by a similar declaration. www.insightsonindia.com 58 Insights IAS INSTA 75 Days REVISION PLAN for Prelims 2020 - InstaTests

66. Consider the following statements regarding Wood’s Despatch 1. It is known as Magna Carta of English education in India 2. It advocated downward filtration theory. 3. It recommended English as the medium of instruction for higher studies and at school level. Which of the statements given above is/are correct? (a) 1 only (b) 2 and 3 only (c) 1 and 2 only (d) 1, 2 and 3

Solution: A

• Lord Macaulay’s Minute (1835) advocated ‘downward filtration theory’. Wood’s Despatch (1854)

• In 1854, Charles Wood prepared a despatch on an educational system for India. Considered the “Magna Carta of English Education in India”, this document was the first comprehensive plan for the spread of education in India. • It asked the government of India to assume responsibility for education of the masses, thus repudiating the ‘downward filtration theory’, at least on paper. • It systematised the hierarchy from vernacular primary schools in villages at bottom, followed by Anglo-Vernacular High Schools and an affiliated college at the district level, and affiliating universities in the presidency towns of Calcutta, Bombay and Madras. • It recommended English as the medium of instruction for higher studies and vernaculars at school level. • It laid stress on female and vocational education, and on teachers’ training. • It laid down that the education imparted in government institutions should be secular. • It recommended a system of grants-in-aid to encourage private enterprise.

67. Consider the following statements regarding Saddler University Commission 1. It held the view that, for the improvement of university education, improvement of secondary education was a necessary pre-condition. 2. Based on its recommendations, the Indian Universities Act was passed in 1904. Which of the statements given above is/are correct?

www.insightsonindia.com 59 Insights IAS INSTA 75 Days REVISION PLAN for Prelims 2020 - InstaTests

(a) 1 only (b) 2 only (c) Both 1 and 2 (d) Neither 1 nor 2

Solution: A

Saddler University Commission (1917-19)

• The commission was set up to study and report on problems of Calcutta University but its recommendations were applicable more or less to other universities also. It reviewed the entire field from school education to university education. It held the view that, for the improvement of university education, improvement of secondary education was a necessary pre-condition. • In 1902, Raleigh Commission was set up to go into conditions and prospects of universities in India and to suggest measures for improvement in their constitution and working. The commission precluded from reporting on primary or secondary education. Based on its recommendations, the Indian Universities Act was passed in 1904.

68. Consider the following statements regarding Central Adoption Resource Authority (CARA) 1. It is a statutory body established under Juvenile Justice Act, 2015. 2. It functions as a nodal body for adoption of Indian children and is mandated to monitor and regulate in-country and inter-country adoption. Which of the statements given above is/are not correct? (a) 1 only (b) 2 only (c) Both 1 and 2 (d) Neither 1 nor 2

Solution: D

Central Adoption Resource Authority (CARA) is a statutory body of Ministry of Women & Child Development established under Juvenile Justice Act, 2015.

www.insightsonindia.com 60 Insights IAS INSTA 75 Days REVISION PLAN for Prelims 2020 - InstaTests

• CARA functions as a nodal body for adoption of Indian children and is mandated to monitor and regulate in-country and inter-country adoption. • It is designated as the Central Authority to deal with inter-country adoptions in accordance with the provisions of the Hague Conventions on Inter-Country Adoptions, 1993, ratified by Government of India in 2003. • CARA primarily deals with adoption of orphan, abandoned and surrendered children through its associated /recognised adoption agencies http://cara.nic.in/about/about_cara.html

69. Consider the following statements regarding Islands Development agency 1. It was constituted in 2017 for preserving the natural eco-system and addressing the security concerns. 2. It is a statutory body constituted under Island Development Act. Which of the statements given above is/are correct? (a) 1 only (b) 2 only (c) Both 1 and 2 (d) Neither 1 nor 2

Solution: A

• The Union Government constituted the Island Development Agency on 1st June, 2017 for the development of islands. • It aims to preserve the natural eco-system and addressing the security concerns. • It is non-statutory body. It is headed by Home Minister. http://pib.nic.in/newsite/PrintRelease.aspx?relid=168895

70. Consider the following statements 1. Bengal Gazette was started by James Augustus Hicky. 2. India Gazette, First Bengali newspaper was started by Harishchandra Ray. Which of the statements given above is/are correct? (a) 1 only (b) 2 only (c) Both 1 and 2 www.insightsonindia.com 61 Insights IAS INSTA 75 Days REVISION PLAN for Prelims 2020 - InstaTests

(d) Neither 1 nor 2

Solution: A

• Bengal Gazette was started by James Augustus Hicky in 1780, Calcutta. • India Gazette was started in 1787. Henry Louis Vivian Derozio was associated with it. • The India Gazette; or, Calcutta Public Advertiser was an English language weekly newspaper published in Kolkata (then Calcutta), the capital of British India. It was the second newspaper printed in India. Founded by Bernard Messink and Peter Reed, two employees, the paper was a strong supporter of the administration of the Governor General Warren Hastings, and a rival to India’s first newspaper Hicky’s Bengal Gazette.

71. Match the following newspaper/journal with their founders 1. Sambad Kaumudi Girish Chandra Ghosh 2. Indian Mirror Devendra Nath Tagore 3. Bande mataram Madam Bikaji Kama 4. Indian Socialist Shyamji Krishna Varma Which of the pairs given above is/are matched correctly? (a) 1, 2 and 3 only (b) 2, 3 and 4 only (c) 1, 3 and 4 only (d) 1, 2, 3 and 4

Solution: B

• Sambad Kaumudi (1821) (Bengali)- Ram Mohan Roy • Mirat-Ul-Akbar (1822)1st Journal in Persian)-Ram Mohan Roy • Banga duta-1822-A weekly in 4 languages.(English,Bengali, Persian,Hindi)Rajaram Mohan Roy, Dwaraka nath Tagore and others. • Jam-i-jahan numah(1822)1st news paper in Urdu- An English Firm

• Indian Mirror-Early 1862-1st English Daily-Devendra Nath Tagore • Som Prakash-1858-Ishwar Chandra Vidyasagar • Bengalee-1862-Girish Chandra Gosh-Taken over by S.N.Benerjea in 1879.

www.insightsonindia.com 62 Insights IAS INSTA 75 Days REVISION PLAN for Prelims 2020 - InstaTests

• Madras mail-1868-1st evening News paper. • Amrita Bazar Patrika-1868-(1st in Bengali later in English)Sisir Kumar Ghosh and Motilal Ghosh • Banga darshana-1873-Bankim chandra chaterjea. • Indina statesman-1875-(Later THE STATESMAN)Robert Knight. • The Hindu-1878-Veera Raghavachari, Subbha rao pandit and G.S.Aiyar • Tribune-1881-Dayal singh Majeetia • Kesari-1881-(Marati) B.G.Tilak • Maharatta-1881-(English)B.G.Tilak • Swadesha mitram—————-G.S.Aiyar • Paridasak-1886-Bipan chandrapal(Publisher)

• New India (Weekly)-Bipin Chandra Pal • New India (Daily)-Annie Besant • Commonweal-Annie Besant • Yugantar-1906-Bhupendranath Datta and Barinder Kumar Ghosh • Sandhya-1906-Brahma Bandhav Upadhyaya • Indian Socialist-(from London)Shyamji Krishna Varma. • Bande mataram—Paris—Madam Bikaji Kama.

72. Which of the following events was/were took place under The Lord Lytton? 1. Vernacular Press Act 2. Ilbert Bill 3. 1st Delhi Durbar Select the correct answer using the code given below (a) 1 and 2 only (b) 2 and 3 only (c) 1 and 3 only (d) 1, 2 and 3

Solution: C

The Lord Lytton 12 April 8 June • Vernacular Press Act, 1878 (1831–1891) 1876 1880 • Second Anglo-Afghan War, (1878–80) • 1st Delhi Durbar out of 3 • Queen Victoria assuming the title of ‘Empress of India’ www.insightsonindia.com 63 Insights IAS INSTA 75 Days REVISION PLAN for Prelims 2020 - InstaTests

The Marquess 8 June 13 December • First Factory Act (1881) of Ripon 1880 1884 • Negotiable Instruments Act (1881) (1827–1909) • Repeal of the Vernacular Press Act (1882) • Ilbert Bill (1883) • Government resolution on local self- government (1882) • Appointment of Education Commission under Sir William Hunter • First complete Census

73. Digital Payment Index has been released by (a) NITI Aayog (b) National Payments Corporation of India (c) Reserve Bank of India (d) Ministry of Electronics and Information Technology

Solution: C

• The Reserve Bank of India (RBI) will issue periodic scores to assess the performance of the digital payments industry on key operating parameters such as rural penetration and innovation in existing modes and channels. • RBI released Digital Payments Index (DPI), the ‘composite’ scoring system will allow both consumers and stakeholders to better gauge local area developments in infrastructure, access, demographic and acceptance related growth with reference to broader domestic and global standards in digital payments.

74. Unani system of medicine was introduced to India during the rule of (a) Delhi Sultanate (b) Mughals (c) Mauryan Kingdom (d) None of the above

Solution: A

www.insightsonindia.com 64 Insights IAS INSTA 75 Days REVISION PLAN for Prelims 2020 - InstaTests

• The term Yūnānī means, as the Perso-Arabic system of medicine was based on the teachings of the Greek physicians Hippocrates and Galen. • The term Unani (Greek) means Perso-Arabic system of medicine. It was introduced to India in 13th century with the establishment of Delhi Sultanate. • Unani system of medicine is the term for Perso-Arabic traditional medicine as practiced in Mughal India and in Muslim culture in South Asia and modern day Central Asia.

75. Which of the following countries shares border with Baltic Sea? 1. Russia 2. Sweden 3. Belarus 4. Norway Select the correct answer using the code given below (a) 2 and 4 only (b) 1 and 2 only (c) 2, 3 and 4 only (d) 1 and 3 only

Solution: B

www.insightsonindia.com 65 Insights IAS INSTA 75 Days REVISION PLAN for Prelims 2020 - InstaTests

DAY – 36 (InstaTest-36)

76. Consider the following statements regarding vacation bench of the Supreme Court 1. It is constituted by the President of India 2. President of India appoints judges for vacation bench. 3. Vacation bench is only allowed to take the cases of grave constitutional issues. Which of the statements given above is/are correct? (a) 1 and 3 only (b) 2 and 3 only (c) 1 and 2 only (d) None

Solution: D

• A Vacation Bench of the Supreme Court is a special bench constituted by the Chief Justice of India for hearing matters during summer vacation or winter holidays. • The Chief Justice may appoint one or more Judges to hear all matters of an urgent nature which under these rules may be heard by a Judge sitting singly. • Litigants can approach the Supreme Court and, if the court decides that the plea is an ‘urgent matter’, the Vacation Bench hears the case on its merits. • During vacations the court generally admits writs related to habeas corpus, certiorari, prohibition and quo warranto matters for enforcement of any fundamental right.

77. Which of the following events took place under the Governor General Lord Curzon 1. Partition of Bengal 2. Surat Split of Congress 3. Establishment of Muslim League Select the correct answer using the code given below (a) 1 only (b) 2 and 3 only (c) 1 and 3 only (d) 1, 2 and 3

www.insightsonindia.com 66 Insights IAS INSTA 75 Days REVISION PLAN for Prelims 2020 - InstaTests

Solution: A

The Lord Curzon of Kedleston • Partition of Bengal (1905) (1859–1925) • Official Secrets Act 1904 to curb free press • 2st Delhi Durbar out of 3 (1903) • Appointment of Police Commission under Sir Andrew Frazer • Appointment of Raleigh University Commission (1902) • Passing of Indian Universities Act 1904 • 2nd Swadeshi Movement (1905-1911) against Partition of Bengal by Lal-Bal-Pal-Aurbindo Ghosh) • Establishment of Archaeological Survey of India • Benaras Hindu Girl’s School was established by Annie Besant in 1904 • (He said, “India is the pivot of our Empire…. If the Empire loses any other part of its Dominion we can survive, but if we lose India, the sun of our Empire will have set.” The Earl of Minto • Morley-Minto Reforms 1909 or The Indian Councils (1845–1914) Act 1909 • Split in Congress in 1907 • Seditious meetings (prohibition) Act 1907 to curb extremist movement • Establishment of Muslim League by Aga Khan III (1906) • Indian Press Act, 1910 • Jamsetji Tata established TISCO in 1907

78. Consider the following statements regarding regional kingdoms 1. The founder of the independent principality of Awadh was Saadat Khan 2. Murshid Kuli Khan was the founder of the independent state of Bengal. Which of the statements given above is/are correct? (a) 1 only (b) 2 only (c) Both 1 and 2 (d) Neither 1 nor 2

Solution: C

www.insightsonindia.com 67 Insights IAS INSTA 75 Days REVISION PLAN for Prelims 2020 - InstaTests

Awadh

• The founder of the independent principality of Awadh was Saadat Khan, popularly known as Burhan-ul-Mulk. Saadat Khan was a Shia. He had joined in a conspiracy against the Sayyid brothers, which resulted in his being given an increased mansab. • Later, driven out of the court, he was prompted to found a new independent state. Saadat Khan committed suicide due to pressure from Nadir Shah who was demanding a huge booty from him. He was succeeded by Safdar Jang as the Nawab of Awadh. Bengal

• Murshid Kuli Khan was the founder of the independent state of Bengal. He was a capable ruler and made Bengal a prosperous state. He was succeeded in 1727 by his son Shujaud-din. His successor, Sarfaraz Khan, was killed in 1740 by Alivardi Khan, the deputy governor of Bihar at Gheria, who assumed power and made himself independent of the Mughal emperor by giving yearly tribute.

79. Consider the following statements regarding National Consumer Disputes Redressal Commission (NCDRC) 1. It is a quasi-judicial commission 2. There is no appellate mechanism against the decision of NCDRC 3. It is headed by a sitting or retired judge of the Supreme Court of India. Which of the statements given above is/are correct? (a) 2 only (b) 1 only (c) 1 and 3 only (d) 1, 2 and 3

Solution: C

The National Consumer Disputes Redressal Commission (NCDRC) is a quasi-judicial commission in India set up in 1988 under the Consumer Protection Act of 1986.

• The commission is headed by a sitting or retired judge of the Supreme Court of India. • According to Section 21 of Consumer Protection Act, 1986, The National Consumer shall have jurisdiction to entertain a complaint valued more than one crore • He/she also have Appellate and Revisional jurisdiction from the orders of State Commissions or the District fora as the case may be.

www.insightsonindia.com 68 Insights IAS INSTA 75 Days REVISION PLAN for Prelims 2020 - InstaTests

• Section 23 of Consumer Protection Act, 1986, provides that any person aggrieved by an order of NCDRC, may prefer an Appeal against such order to Supreme Court of India within a period of 30 days.

80. Chatuspathis or Tols were referred to (a) Village accountants (b) Nomads and herders. (c) Head of trade associations. (d) Centers of higher education

Solution: D

• Chatuspathis or Tols, as they were called in Bihar and Bengal, were the centres of higher education. Some of the famous centres for Sanskrit education were Kasi (Varanasi), Tirhut (Mithila), Nadia and Utkala. Madrasahs were the institutions of higher learning for Persian and Arabic, Persian being the court language and learnt by the Muslims as well as the Hindus. Azimabad (Patna) was a famous centre for Persian education. People interested in the study of the Quran and Muslim theology had to acquire proficiency in Arabic.

81. Consider the following statements regarding Dual Government in Bengal 1. Warren Hasting introduced the dual system of government. 2. Diwani and nizamat came under the control of the Company. 3. The nawab was responsible for maintaining peace and order, but he was depended on Company Which of the statements given above is/are correct? (a) 2 only (b) 2 and 3 only (c) 1 and 3 only (d) 1, 2 and 3

Solution: B

www.insightsonindia.com 69 Insights IAS INSTA 75 Days REVISION PLAN for Prelims 2020 - InstaTests

Dual Government in Bengal (1765-72)

• After the battle of Buxar, the East India Company became the real masters of Bengal. Robert Clive introduced the dual system of government, i.e., the rule of the two— the Company and the Nawab—in Bengal in which both the diwani, i.e., collecting revenues, and nizamat, i.e., police and judicial functions, came under the control of the Company. The Company exercised diwani rights as the diwan and the nizamat rights through its right to nominate the deputy subahdar. The Company acquired the diwani functions from the emperor and nizamat functions from the subahdar of Bengal. • The system held a great advantage for the Company. It left the appearance of authority to the puppet Indian ruler, while keeping the sovereign power in the hands of the Company. The nawab was responsible for maintaining peace and order, but he depended both for funds and forces upon the Company because the latter controlled the army and revenues.

82. Consider the following statements regarding Antrix 1. It is the commercial arm of Indian Space Research Organisation 2. It provides launch services for customer satellites Which of the statements given above is/are correct? (a) 1 only (b) 2 only (c) Both 1 and 2 (d) Neither 1 nor 2

Solution: C

Antrix is the commercial arm of Indian Space Research Organisation (ISRO). Antrix Corporation Limited (Antrix) was incorporated on 1992 under the Companies Act, 1956, to market the products and services of the Indian Space Research Organisation (ISRO).

• It is a wholly Government of India owned Company under the administrative control of Department of Space (DOS). • In the year 2008, the Company was awarded ‘MINIRATNA’ status. Current business activities of Antrix • Provisioning of communication satellite transponders to various users • Providing launch services for customer satellites • Marketing of data from Indian and foreign remote sensing satellites • Building and marketing of satellites as well as satellite sub-systems www.insightsonindia.com 70 Insights IAS INSTA 75 Days REVISION PLAN for Prelims 2020 - InstaTests

• Establishing ground infrastructure for space applications • Mission support services for satellites. http://www.antrix.co.in/about-us

83. Consider the following statements regarding Dadabhai Naoroji 1. He served as the first Indian member of the British parliament. 2. He was Congress president thrice. 3. He became a member of the royal commission on Indian expenditure. Which of the statements given above is/are correct? (a) 1 and 2 only (b) 2 and 3 only (c) 1 and 3 only (d) 1, 2 and 3

Solution: D

Dadabhai Naoroji

• Dadabhai Naoroji, the “Grand Old Man of India”, who was among the first leaders who stirred national consciousness in the country. • Born in 1825 at Navsari, in present-day Gujarat. Key contributions:

• He was closely involved with the Indian National Congress in its early phase. • He served as the first Indian member of the British parliament. • His first agitation, in 1859, concerned recruitment to the Indian Civil Service. • In 1865 and 1866, Naoroji helped found the London Indian Society and the East India Association. The two organisations sought to bring nationalist Indians and sympathetic Britons on one platform. • As the secretary of the East India Association, Naoroji travelled in India to gather funds and raise national awareness. • In 1885, Naoroji became a vice-president of the Bombay Presidency Association, was nominated to the Bombay legislative council by Governor Lord Reay, and helped form the Indian National Congress. • He was Congress president thrice, in 1886, 1893, and 1906. • In 1893, he helped form an Indian parliamentary committee to attend to Indian interests. • In 1895, he became a member of the royal commission on Indian expenditure. www.insightsonindia.com 71 Insights IAS INSTA 75 Days REVISION PLAN for Prelims 2020 - InstaTests

• Dadabhai Naoroji was among the key proponents of the ‘Drain Theory’, disseminating it in his 1901 book ‘Poverty and Un-British Rule in India’. What is Drain Theory?

• Imperial Britain was draining away India’s wealth to itself through exploitative economic policies, including India’s rule by foreigners; the heavy financial burden of the British civil and military apparatus in India; the exploitation of the country due to free trade; non-Indians taking away the money that they earned in India; and the interest that India paid on its public debt held in Britain.

84. Consider the following statements regarding Wahabi Movement 1. The Wahabi Movement was essentially an Islamic revivalist movement founded by Syed Ahmed 2. Syed Ahmed condemned the western influence on Islam and advocated a return to pure Islam 3. The Wahabis played an important role in spreading Pro-British sentiments. Which of the statements given above is/are correct? (a) 1 and 2 only (b) 2 only (c) 1 and 3 only (d) 1, 2 and 3

Solution: A

Wahabi Movement

• The Wahabi Movement was essentially an Islamic revivalist movement founded by Syed Ahmed of Rai Bareilly who was inspired by the teachings of Abdul Wahab (1703- 87) of Saudi Arabia and Shah Waliullah of Delhi. Syed Ahmed condemned the western influence on Islam and advocated a return to pure Islam and society as it was in the Arabia of the Prophet’s time. • Syed Ahmed was acclaimed as the desired leader (Imam). A countrywide organisation with an elaborate secret code for its working under spiritual vice-regents (Khalifas) was set up, and Sithana in the north-western tribal belt was chosen as a base for operations. In India, its important centre was at Patna though it had its missions in Hyderabad, Madras, Bengal, United Provinces and Bombay. Since Dar-ul- Harb (the land of kafirs) was to be converted into Dar-ul-Islam (the land of Islam), a jihad was declared against the Sikh kingdom of Punjab. After the defeat of the Sikh

www.insightsonindia.com 72 Insights IAS INSTA 75 Days REVISION PLAN for Prelims 2020 - InstaTests

ruler and incorporation of Punjab into the East India Company’s dominion in 1849, the English dominion in India became the sole target of the Wahabis’ attacks. • The Wahabis played an important role in spreading anti-British sentiments. A series of military operations by the British in the 1860s on the Wahabi base in Sithana and various court cases of sedition on the Wahabis weakened the Wahabi resistance, although sporadic encounters with the authorities continued into the 1880s and 1890s.

85. Which of the following rights is/are entitled to the farmers by Protection of Plant Variety and Farmers Right Act, 2001 1. Farmers are entitled to save, use, sow, re-sow, exchange, share or sell their farm produce, including seed of protected varieties 2. Plant breeders and legal entities including farmers who provide Plant Genetic Resources (PGR) to breeders for developing new varieties shall receive a fair share of benefit from the commercial gains 3. Farmers have the right to access seed of registered varieties at a reasonable and remunerative price. Select the correct answer using the code given below (a) 2 and 3 only (b) 2 only (c) 1 and 3 only (d) 1, 2 and 3

Solution: D

About Protection of Plant Variety and Farmers Right Act, 2001

• The Protection of Plant Varieties and Farmers’ Rights Act (PPV&FR Act) seeks to address the rights of plant breeders and farmers on an equal footing. It affirms the necessity of recognizing and protecting the rights of farmers with respect to the contribution they make in conserving, improving and making Plant Genetic Resources (PGR) available for the development of new plant varieties. Right 1: Access to seed [Section 39(1)(iv)]

• Farmers are entitled to save, use, sow, re-sow, exchange, share or sell their farm produce, including seed of protected varieties, in the same manner as they were entitles to before the coming into force to the PPV&FR Act. However, farmers are not entitled to sell branded seed of a variety protected under this Act. Farmers can use farm saved seed from a crop cultivated in their own. www.insightsonindia.com 73 Insights IAS INSTA 75 Days REVISION PLAN for Prelims 2020 - InstaTests

Right 2: Benefit sharing [Section 26]

• Plant breeders and legal entities including farmers who provide Plant Genetic Resources (PGR) to breeders for developing new varieties shall receive a fair share of benefit from the commercial gains of the registered varieties. Out of all the national plant variety protection laws enacted since 2001, the PPV&FR Act is the first that integrates a provision for access and benefit-sharing (ABS) along with Plant Breeder’s Rights (PBRs). Accession of the genetic resource used in breeding is permitted under the Biological Diversity Act, 2002. However, the PPV&FR Act requires a breeder to make a sworn declaration on the geographical origin of the genetic resources used in the pedigree of the new variety, and its accession. Right 3: Compensation [Section 39(2)]

• Registered seed must be sold with the full disclosure of their agronomic performance under recommended management conditions. When such seed is sold to farmers but fails to provide the expected performance under recommended management conditions, the farmer is eligible to claim compensation from the breeder through the intervention of the PPV&FR Authority. https://vikaspedia.in/agriculture/policies-and-schemes/crops-related/protection-of-plant- varieties-and-rights-of-farmers/farmers2019-rights-in-the-ppv-fr-act-2001

86. Consider the following statements regarding women organization 1. Sarla Devi Chaudhurani convened the first meeting of the Bharat Stree Mahamandal. 2. Mehribai Tata founded the Ladies Social Conference. 3. Pandita Ramabai Saraswati founded the Arya Mahila Samaj. Which of the statements given above is/are correct? (a) 1 only (b) 2 and 3 only (c) 1 and 3 only (d) 1, 2 and 3

Solution: C

• In 1910, Sarla Devi Chaudhurani convened the first meeting of the Bharat Stree Mahamandal in Allahabad. • Considered as the first major Indian women’s organisation set up by a woman, its objectives included promotion of education for women, abolition of the purdah

www.insightsonindia.com 74 Insights IAS INSTA 75 Days REVISION PLAN for Prelims 2020 - InstaTests

system and improvement in the socio-economic and political status of woman all over India. Sarla Devi believed that the man working for women’s upliftment lived ‘under the shade of Manu’. • Ramabai Ranade founded the Ladies Social Conference (Bharat Mahila Parishad), under the parent organization National Social Conference, in 1904 in Bombay. • Pandita Ramabai Saraswati founded the Arya Mahila Samaj to serve the cause of women. She pleaded for improvement in the educational syllabus of Indian women before the English Education Commission which was referred to Queen Victoria. This resulted in medical education for women which started in Lady Dufferin College. Later Ramabai Ranade established a branch of Arya Mahila Samaj in Bombay. • In 1925, the National Council of Women in India, a national branch of the International Council of Women, was formed. Mehribai Tata played a vital role in its formation and advancement.

87. Consider the following pairs of political associations and their founders Political Associations Founders 1. The Madras Mahajan : M. Viraraghavachari, Sabha B. Subramaniya Aiyer and P. Anandacharlu 2. The Bombay Presidency : Mahadeo Govind Association Ranade 3. The Poona Sarvajanik : Badruddin Tyabji, Sabha Pherozshah Mehta and K.T. Telang Which of the pairs given above is/are correctly matched? (a) 1 only (b) 2 and 3 only (c) 1 and 3 only (d) 1, 2 and 3

Solution: A

Political Associations in Bombay

• The Poona Sarvajanik Sabha was founded in 1867 by Mahadeo Govind Ranade and others, with the object of serving as a bridge between the government and the people. • The Bombay Presidency Association was started by Badruddin Tyabji, Pherozshah Mehta and K.T. Telang in 1885.

www.insightsonindia.com 75 Insights IAS INSTA 75 Days REVISION PLAN for Prelims 2020 - InstaTests

Political Associations in Madras

• The Madras Mahajan Sabha was founded in 1884 by M. Viraraghavachari, B. Subramaniya Aiyer and P. Anandacharlu.

88. B N Srikrishna committee, sometime seen in the news, is related to (a) Resolving Kashmir Issue (b) Universal Basic Income (c) Data Protection (d) None of the above

Solution: C

• Cleared by the Cabinet, the Personal Data Protection Bill is due to be placed in Parliament. • The Bill has three key aspects that were not previously included in a draft version, prepared by a committee headed by retired Justice B N Srikrishna. How does the bill seek to regulate data? The bill constitutes 3 personal information types: 1. Critical 2. Sensitive 3. General 1. Sensitive data constitutes or is related to passwords, financial data, health data, official identifier, sexual orientation, religious or caste data, biometric data and genetic data. It may be processed outside India with the explicit consent of the user. 2. Critical data will be characterised by the government every once in a while, and must be stored and handled only in India. 3. General data: Any data that is non-critical and non-sensitive is categorised as general data with no limitation on where it is stored or managed. Other Key provisions:

• Data principal: As per the bill, it is the individual whose data is being stored and processed. • Exemptions: The government is qualified to order any data fiduciary to acquire personal and non-personal/anonymised data for the sake of research and for national security and criminal investigations. www.insightsonindia.com 76 Insights IAS INSTA 75 Days REVISION PLAN for Prelims 2020 - InstaTests

• Social media companies, which are deemed significant data fiduciaries based on factors such as volume and sensitivity of data as well as their turnover, should develop their own user verification mechanism. • An independent regulator Data Protection Agency (DPA) will oversee assessments and audits and definition making. • Each company will have a Data Protection Officer (DPO) who will liaison with the DPA for auditing, grievance redressal, recording maintenance and more. • The bill also grants individuals the right to data portability, and the ability to access and transfer one’s own data. • The right to be forgotten: this right allows an individual to remove consent for data collection and disclosure. https://www.insightsonindia.com/2019/12/07/data-protection-bill/

89. Consider the following statements regarding Lucknow Session (1916) 1. It was presided over by Ambika Charan Majumdar. 2. Readmission of Extremists to Congress with the efforts of Annie Besant and Tilak. 3. Lucknow Pact between Congress and Muslim League, led to acceptance of separate electorate. Which of the statements given above is/are correct? (a) 1 only (b) 2 and 3 only (c) 1 and 3 only (d) 1, 2 and 3

Solution: D

Lucknow Session of the Indian National Congress (1916) Readmission of Extremists to Congress The Lucknow session of the Indian National Congress, presided over by a Moderate, Ambika Charan Majumdar, finally readmitted the Extremists led by Tilak to the Congress fold. Various factors facilitated this reunion: 1. Old controversies had become meaningless now. 2. Both the Moderates and the Extremists realized that the split had led to political inactivity.

www.insightsonindia.com 77 Insights IAS INSTA 75 Days REVISION PLAN for Prelims 2020 - InstaTests

3. Annie Besant and Tilak had made vigorous efforts for the reunion. To allay Moderate suspicions, Tilak had declared that he supported a reform of administration and not an overthrow of the government. He also denounced acts of violence. 4. The death of two Moderates, Gokhale and Pherozshah Mehta, who had led the Moderate opposition to the Extremists, facilitated the reunion. Lucknow Pact between Congress and Muslim League

• Another significant development to take place at Lucknow was the coming together of the Muslim League and the Congress and the presentation of common demands by them to the government. This happened at a time when the Muslim League, now dominated by the younger militant nationalists, was coming closer to the Congress objectives and turning increasingly anti-imperialist.

90. Consider the following statements regarding The Tolstoy Farm 1. The Tolstoy Farm was the first of its kind established by Gandhi. 2. It was to house the families of the satyagrahis. Which of the statements given above is/are correct? (a) 1 only (b) 2 only (c) Both 1 and 2 (d) Neither 1 nor 2

Solution: B

• As it became rather difficult to sustain the high pitch of the struggle, Gandhi decided to devote all his attention to the struggle. The Tolstoy Farm was founded in 1910 and named as such by Gandhi’s associate, Herman Kallenbach, after the Russian writer and moralist, whom Gandhi admired and corresponded with. Besides being an experiment in education, it was to house the families of the satyagrahis and to give them a way to sustain themselves. • The Tolstoy Farm was the second of its kind established by Gandhi. He had set up the Phoenix Farm in 1904 in Natal, inspired by a reading of John Ruskin’s Unto This Last, a critique of capitalism, and a work that extolled the virtues of the simple life of love, labour, and the dignity of human beings. As at the Phoenix settlement, at Tolstoy Farm too, manual work went hand-in-hand with instruction. Vocational training was introduced to give “all-round development to the boys and girls”.

www.insightsonindia.com 78 Insights IAS INSTA 75 Days REVISION PLAN for Prelims 2020 - InstaTests

91. Consider the following statements regarding Red Panda: 1. They are mainly found in the forests of India, Nepal, Bhutan and and China. 2. It has been listed as near threatened in IUCN’s Red list. 3. It is the state animal of Meghalaya. Which of the statements given above is/are correct? (a) 1 only (b) 3 only (c) 2 and 3 only (d) 1 and 2 only

Solution: A

• The red panda is a small arboreal mammal found in the forests of India, Nepal, Bhutan and the northern mountains of Myanmar and southern China. • The red panda is slightly larger than a domestic cat with a bear-like body and thick russet fur. The belly and limbs are black, and there are white markings on the side of the head and above its small eyes. Red pandas are very skillful and acrobatic animals that predominantly stay in trees. Almost 50% of the red panda’s habitat is in the Eastern Himalayas. It is the state animal of Sikkim. • Clouded Leopard is the state animal of Meghalaya.

www.insightsonindia.com 79 Insights IAS INSTA 75 Days REVISION PLAN for Prelims 2020 - InstaTests

92. Consider the following statements regarding Jallianwala Bagh Massacre 1. Following the incident, Rabindranath Tagore renounced his knighthood in protest. 2. Gandhi gave up the title of Kaiser-i-Hind following the incident. 3. Udham Singh assassinated Michael O’Dwyer, the Lieutenant-Governor who presided over the brutal British suppression of the 1919 protests in Punjab. Which of the statements given above is/are correct? (a) 1 and 2 only (b) 2 and 3 only (c) 1 only (d) 1, 2 and 3

Solution: D

Jallianwala Bagh Massacre (April 13, 1919)

• Amritsar was the worst affected by violence. In the beginning there was no violence by the protestors. Indians shut down their shops and normal trade and the empty streets showed the Indians’ displeasure at the British betrayal. On April 9, two nationalist leaders, Saifuddin Kitchlew and Dr Satyapal, were arrested by the British officials without any provocation except that they had addressed protest meetings, and taken to some unknown destination. This caused resentment among the Indian protestors who came out in thousands on April 10 to show their solidarity with their leaders. Soon the protests turned violent because the police resorted to firing in which some of the protestors were killed. Tension ran high. • In the riot that followed, five Englishmen are reported to have been killed and Marcella Sherwood, an English woman missionary going on a bicycle, was beaten up. • On Baisakhi day, a large crowd of people mostly from neighbouring villages, unaware of the prohibitory orders in the city, gathered in the Jallianwala Bagh, a popular place for public events, to celebrate the Baisakhi festival. Local leaders had also called for a protest meeting at the venue. • The entire nation was stunned. Rabindranath Tagore renounced his knighthood in protest. Gandhi gave up the title of Kaiser-i-Hind, bestowed by the British for his work during the Boer War. Gandhi was overwhelmed by the atmosphere of total violence and withdrew the movement on April 18, 1919. • Udham Singh, who bore the name, Ram Mohammad Singh Azad, later assassinated Michael O’Dwyer, the Lieutenant-Governor who presided over the brutal British suppression of the 1919 protests in Punjab. Udham Singh was hanged in 1940 for his deed. (His ashes were returned to India in 1974.)

www.insightsonindia.com 80 Insights IAS INSTA 75 Days REVISION PLAN for Prelims 2020 - InstaTests

93. Consider the following statements regarding Hindustan Republican Association 1. The HRA was founded by Ramprasad Bismil, Jogesh Chandra Chatterjee and Sachin Sanyal. 2. Under the sole leadership of Bhagat Singh, the name of HRA was changed to Hindustan Socialist Republican Association (HSRA). Which of the statements given above is/are correct? (a) 1 only (b) 2 only (c) Both 1 and 2 (d) Neither 1 nor 2

Solution: A

• The revolutionary activity in this region was dominated by the Hindustan Republican Association/Army or HRA (later renamed Hindustan Socialist Republican Association or HSRA). The HRA was founded in October 1924 in Kanpur by Ramprasad Bismil, Jogesh Chandra Chatterjee and Sachin Sanyal, with an aim to organise an armed revolution to overthrow the colonial government and establish in its place the Federal Republic of United States of India whose basic principle would be adult franchise.

The HSRA

• Determined to overcome the Kakori setback, the younger revolutionaries, inspired by socialist ideas, set out to reorganize Hindustan Republic Association at a historic meeting in the ruins of Ferozshah Kotla in Delhi (September 1928). Under the leadership of Chandra Shekhar Azad, the name of HRA was changed to Hindustan Socialist Republican Association (HSRA). The participants included Bhagat Singh, Sukhdev, Bhagwaticharan Vohra from Punjab and Bejoy Kumar Sinha, Shiv Verma and Jaidev Kapur from the United Provinces. The HSRA decided to work under a collective leadership and adopted socialism as its official goal.

94. Consider the following statements regarding Katchatheevu islands: 1. It is located in the Palk Strait. 2. It was ceded to Sri Lanka through the Indo-Sri Lanka Peace Accord in 1989. 3. St. Antony’s shrine is located on the island. Which of the statements given above is/are correct? (a) 1 and 3 only www.insightsonindia.com 81 Insights IAS INSTA 75 Days REVISION PLAN for Prelims 2020 - InstaTests

(b) 2 only (c) 1, 2 and 3 (d) 3 only

Solution: A

• Katchatheevu is an uninhabited islet in the Palk Strait. The island was earlier part of the Ramnad Kingdom which existed in Madurai district of India. Later on with the British rule on the Indian subcontinent was the island part of the Madras Presidency. • In 1974, Katchatheevu was ceded to Sri Lanka through the Indo-Sri Lankan Maritime agreement to settle the maritime boundary in the Palk Strait. • In 1976 through another accord, India further gave up its fishing rights in the region. • St. Antony’s shrine is the only structure on the island. It is a church named after St. Antony, considered a patron-saint of seafarers by the Christians. It was built by a prosperous Indian an Indian Catholic (Tamilian) fisherman Srinivasa Padaiyachi in the early 20th century. The annual church festival runs for three days. The priests from both India and Sri Lanka conduct the mass and car procession. Pilgrims from India are ferried mostly Rameswaram. According to the agreement between the Indian and Sri Lankan government, the citizens of India are not required to possess an Indian passport or Sri Lankan visa for visiting Kachchatheevu.

www.insightsonindia.com 82 Insights IAS INSTA 75 Days REVISION PLAN for Prelims 2020 - InstaTests https://www.thehindu.com/news/cities/Madurai/allow-country-boats-to-katchatheevu- church-festival/article30667291.ece

95. Consider the following statements regarding education reforms 1. The Calcutta Madrasah was established by Jonathan Duncan 2. The Sanskrit College was established by Warren Hastings Which of the statements given above is/are correct? (a) 1 only (b) 2 only (c) Both 1 and 2 (d) Neither 1 nor 2

Solution: D

• The Calcutta Madrasah was established by Warren Hastings in 1781 for the study of Muslim law and related subjects. • The Sanskrit College was established by Jonathan Duncan, the resident, at Benaras in 1791 for study of Hindu law and philosophy. • Fort William College was set up by Wellesley in 1800 for training of civil servants of the Company in languages and customs of Indians (closed in 1802). • The Calcutta Madrasah and the Sanskrit College were designed to provide a regular supply of qualified Indians to help the administration of law in the Company’s court, and the knowledge of classical languages and vernaculars was useful in correspondence with Indian states.

96. Consider the following statements regarding the Hydroxychloroquine (HCQ): 1. It is used to prevent and treat malaria in areas where malaria remains sensitive to chloroquine. 2. It is listed on the World Health Organization’s List of Essential Medicines, 3. It is used in the treatment of rheumatoid arthritis. Which of the statements given above is/are correct? (a) 1 and 2 only (b) 3 only (c) 2 and 3 only

www.insightsonindia.com 83 Insights IAS INSTA 75 Days REVISION PLAN for Prelims 2020 - InstaTests

(d) 1, 2 and 3

Solution: D

• Hydroxychloroquine (HCQ), sold under the brand name Plaquenil among others, is a medication used to prevent and treat malaria in areas where malaria remains sensitive to chloroquine. Other uses include treatment of rheumatoid arthritis, lupus, and porphyria cutanea tarda. It is taken by mouth. It is also being studied as a treatment for coronavirus disease 2019 (COVID-19). https://m.economictimes.com/industry/healthcare/biotech/pharmaceuticals/india-will- allow-some-exports-of-anti-malaria-drug-after-trump-appeal/articleshow/75022468.cms

97. Consider the following pairs of the British officers who resisted the 1857 revolt and respective revolt centres: Revolt Centre British Officer 1. Delhi : John Nicholson 2. Kanpur : Sir Colin Campbell 3. Jhansi : Sir Hugh Rose 4. Benaras : Colonel James Neill Which of the pairs given above is/are correctly matched? (a) 1, 2 and 3 only (b) 2, 3 and 4 only (c) 1, 3 and 4 only (d) 1, 2, 3 and 4

Solution: D

• Indian Mutiny, also called Sepoy Mutiny or First War of Independence, widespread but unsuccessful rebellion against British rule in India in 1857–59. Begun in Meerut by Indian troops (sepoys) in the service of the British East India Company, it spread to Delhi, Agra, Kanpur, and Lucknow. It is often called the First War of Indian Independence and other similar names.

www.insightsonindia.com 84 Insights IAS INSTA 75 Days REVISION PLAN for Prelims 2020 - InstaTests

The British Resistance to 1857 revolt Revolt Centre and British officers who led the British troops are as follows: 1. Delhi – Lieutenant Willoughby, John Nicholson, Lieutenant Hudson 2. Kanpur – Sir Hugh Wheeler, Sir Colin Campbell 3. Lucknow – Henry Lawrence, Brigadier Inglis, Henry Havelock, James Outram, Sir Colin Campbell 4. Jhansi – Sir Hugh Rose 5. Benaras – Colonel James Neill

98. Which of the following organizations releases the Gender Social Norms Index (GSNI)? (a) World Bank (b) World Economic Forum (WEF) (c) The United Nations Educational, Scientific and Cultural Organization (UNESCO) (d) The United Nations Development Programme (UNDP)

Solution: D

• UNDP releases the Gender Social Norms Index (GSNI). The Gender Social Norms Index (GSNI) measures how social beliefs obstruct gender equality in areas like politics, work, and education, and contains data from 75 countries, covering over 80 percent of the world’s population. http://hdr.undp.org/en/gsni

99. Consider the following statements regarding Tebhaga Movement 1. Flood Commission had recommended one-half share to the bargardars and share- croppers. 2. Muslims participated in large numbers in this movement. Which of the statements given above is/are correct? (a) 1 only (b) 2 only (c) Both 1 and 2 (d) Neither 1 nor 2

www.insightsonindia.com 85 Insights IAS INSTA 75 Days REVISION PLAN for Prelims 2020 - InstaTests

Solution: B

Tebhaga Movement

• In September 1946, the Bengal Provincial Kisan Sabha gave a call to implement, through mass struggle, the Flood Commission recommendations of tebhaga—two- thirds’ share—to the bargardars, the share-croppers also known as bagchasi or adhyar, instead of the one-half share. The bargardars worked on lands rented from the jotedars. The communist cadres, including many urban student militias went to the countryside to organise the bargardars. The central slogan was “nij khamare dhan tolo”—i.e., sharecroppers taking the paddy to their own threshing floor and not to the jotedar’s house, as before, so as to enforce tebhaga. • The storm centre of the movement was north Bengal, principally among Rajbanshis— a low caste of tribal origin. • Muslims also participated in large numbers. The movement dissipated soon, because of the League ministry’s sop of the Bargardari Bill, an intensified repression, the popularization of the Hindu Mahasabha’s agitation for a separate Bengal and renewed riots in Calcutta which ended the prospects of sympathetic support from the urban sections.

100. Consider the following statements regarding Kurzarbeit scheme/policy: 1. It was rolled out by the Brazil government during 2008-09 global economic crisis. 2. It aims to address workers who are impacted by loss of income due to shortened work hours during economic crisis. Which of the statements given above is/are correct? (a) 1 only (b) 2 only (c) Both 1 and 2 (d) Neither 1 nor 2

Solution: B

• Amid the all-round disruption caused to the economy by the novel coronavirus outbreak, a concern across the world is the possibility of loss of jobs. Various governments have unveiled various measures to address such concerns, and one of the most talked about is Kurzarbeit, Germany’s existing scheme that provides for

www.insightsonindia.com 86 Insights IAS INSTA 75 Days REVISION PLAN for Prelims 2020 - InstaTests

partial compensation for a worker’s earnings in such situations, and now modified to account for the current crisis. • The policy was rolled out during the 2008 economic crisis while its origins date back as far as the early 20th century, before and after World War I. • When companies face a loss of earnings due to unforeseen economic situations, they often need to cut back on their working hours or send some of their employees home. The Kurzarbeit scheme aims to address workers who are impacted by loss of income due to shortened work hours during such times. They can apply for short-term work benefits under the scheme, with the government stepping in to pay employees a part of their lost income. This helps the companies retain their employees instead of laying them off, and allows the latter to sustain themselves for a period of up to 12 months. https://indianexpress.com/article/explained/explained-germanys-kurzarbeit-scheme-for- payment-of-staff-hit-by-lost-work-hours-6329831/

www.insightsonindia.com 87 Insights IAS INSTA STORIE S

😊

www.insightsonindia.com INSIGHTS IAS 

www.insightsonindia.com INSIGHTS IAS